Exam Simulation - Test 2 Flashcards

1
Q

Dr. Andy Amato works as a psychotherapist at a community mental health clinic and has just tested positive for the HIV virus. In terms of ethical requirements, Dr. Amato:
Select one:

A.
should inform his clients of his medical condition as soon as possible.

B.
should inform his clients and employer of his medical condition as soon as possible.

C.
should refrain from initiating any professional activities that might be adversely affected by his medical condition.

D.
is not obligated to take any particular action in this situation.

A

Standard 2.06 of the APA’s Ethics Code and Principle II.11 of the Canadian Code of Ethics for Psychologists apply to this situation.
a. Incorrect See explanation for response c.

b. Incorrect See explanation for response c.
c. CORRECT This answer is most consistent with ethical guidelines. For example, Standard 2.06 requires psychologists to “refrain from initiating an activity when they know or should know that there is a substantial likelihood that their personal problems will prevent them from performing their work-related activities in a competent manner.” Personal problems include health-related issues that might compromise a psychologist’s job performance.
d. Incorrect See explanation for response c.

The correct answer is: should refrain from initiating any professional activities that might be adversely affected by his medical condition.

How well did you know this?
1
Not at all
2
3
4
5
Perfectly
2
Q

Dr. Aimee A. is in private practice and, based on her experience and the experience of her colleagues, she has concluded that clients with signs of Malingering are at high risk for suing their therapists for malpractice. As a result, Dr. A. has decided to refer clients who exhibit signs of Malingering during the first therapy session to another therapist she knows is willing to work with them. Dr. A’s practice is:
Select one:

A.
unethical.

B.
ethical only if she makes her policy about decisions related to accepting clients clear at the outset of therapy.

C.
ethical only if she makes the referral before a therapeutic relationship has been established with the client.

D.
ethical.

A

Although this issue isn’t directly addressed by ethical guidelines, the guidelines permit (or recommend) termination of services under certain circumstances [see, e.g., Standard 10.10 of the APA’s Ethics Code and Principle I and II (Values) and Principles II.34 and II.37 of the Canadian Code of Ethics].
a. Incorrect See explanation for response d.

b. Incorrect See explanation for response d.
c. Incorrect See explanation for response d.
d. CORRECT Psychologists in private practice do not have an ethical or legal duty to provide professional services to individuals they do not want to serve. Therefore, it would be acceptable to make the decision to refer a client who is exhbiting signs of Malingering after an intial therapy session. (Ethical guidelines require psychologists not to discriminate on the basis of race, gender, or any other basis proscribed by law, but litigious clients do not fall into this category.)

The correct answer is: ethical.

How well did you know this?
1
Not at all
2
3
4
5
Perfectly
3
Q

: Which of the following techniques would be useful for predicting status on a single dichotomous criterion using two or more predictors when the assumptions for a discriminant analysis are not met?
Select one:

A.
canonical correlation

B.
path analysis

C.
logistic regression

D.
multiple regression

A

For the exam, you want to be familiar with the techniques listed in the answers to this questions. Information about these techniques is provided in the Statistics and Research Design chapter of the written study materials.
a. Incorrect See explanation for response c.

b. Incorrect See explanation for response c.
c. CORRECT Logistic regression is considered more flexible than discriminanant analysis since it can be used when the data violate some of the assumptions of discriminant analysis (e.g., the assumptions of linearity, normality, and/or homogeneity of variances).
d. Incorrect See explanation for response c.

The correct answer is: logistic regression

How well did you know this?
1
Not at all
2
3
4
5
Perfectly
4
Q

Most children do not understand that death is irreversible and universal until about _____ years of age.
Select one:

A.
four

B.
seven

C.
ten

D.
thirteen

A

Research has found that children typically progress through predictable stages in acquiring an understanding of death.

a. Incorrect See explanation for answer c
b. Incorrect See explanation for answer c.
c. CORRECT The age at which children understand that death is irreversible and universal varies, depending on certain factors. In general, however, most children achieve this understanding at about ten years of age.

d. Incorrect See explanation for answer c.
The correct answer is: ten

How well did you know this?
1
Not at all
2
3
4
5
Perfectly
5
Q

Before conducting a court-ordered evaluation of a minor, you should:
Select one:

A.
obtain an informed consent from the minor.

B.
obtain an informed consent from the minor’s legal guardian.

C.
discuss the purpose of the evaluation with the minor.

D.
make sure the minor’s legal guardian is present during the evaluation.

A

A court-ordered evaluation precludes the need for an informed consent, even when the person to be evaluated is a minor.
a. Incorrect See explanation for response c.

b. Incorrect See explanation for response c.
c. CORRECT This response is most consistent with the provisions of the Specialty Guidelines for Forensic Psychology (APA, 2012), which states that the subject of a court-ordered evaluation must be informed of the evaluation nature and purpose.
d. Incorrect See explanation for response c.

The correct answer is: discuss the purpose of the evaluation with the minor.

How well did you know this?
1
Not at all
2
3
4
5
Perfectly
6
Q

According to Herbert Simon (1960), “bounded discretion”:
Select one:

A.
is due to limited cognitive ability.

B.
ensures that the optimal alternative is chosen.

C.
limits a decision-maker’s choices.

D.
reduces a leader’s effectiveness in ambiguous situations.

A

Knowing that Simon distinguished between two models of individual decision-making (rational-economic and bounded rationality) would have helped you identify the correct answer to this question.
a. Incorrect See explanation for response c.

b. Incorrect See explanation for response c.
c. CORRECT Bounded discretion is also known as bounded rationality and refers to the limitations on a decision-maker’s choices resulting from certain social, legal, moral, and organizational factors.
d. Incorrect See explanation for response c.

The correct answer is: limits a decision-maker’s choices.

How well did you know this?
1
Not at all
2
3
4
5
Perfectly
7
Q

When faced with an “undifferentiated family ego mass,” a family therapist who has adopted a Bowenian approach would be most likely to do which of the following?
Select one:

A.
Help differentiate family members by becoming emotionally triangulated into family dyads.

B.
Help differentiate family members by using behavioral and cognitive-behavioral techniques that promote individuation.

C.
Work first with the most differentiated family member to help him or her achieve greater individuation.

D.
Work first with the “identified patient” to help him or her become more differentiated.

A

Bowenian therapists often work initially with the most mature and differentiated family.
a. Incorrect When working with two family members (usually the spouses), a Bowenian therapist becomes the third member in a therapeutic (not an emotional) triangle. Emotional triangles are a sign of dysfunction in a family.

b. Incorrect Bowenian therapists do not typically use behavioral or cognitive-behavioral techniques.
c. CORRECT Ideally, the increased individuation of the most differentiated family member will motive other family members to become less emotionally involved in the family.
d. Incorrect A Bowenian would work only with the identified patient if he/she were also the most differentiated family member, which is often not the case.

The correct answer is: Work first with the most differentiated family member to help him or her achieve greater individuation.

How well did you know this?
1
Not at all
2
3
4
5
Perfectly
8
Q

Adequate “floor” is especially important when a test will be used to:
Select one:

A.
make intra-individual comparisons.

B.
confirm a diagnosis of Intellectual Disability.

C.
assign students to advanced placement classes.

D.
evaluate the readiness of employees for promotion.

A

In the context of testing, “floor” refers to the test’s ability to discriminate among examinees at the low end of the score range.
a. Incorrect See explanation for response b.

b. CORRECT To make a diagnosis of Intellectual Disability and distinguish between different levels of severity, you have to be able to discriminate among individuals with low intelligence. This requires adequate “floor” - i.e., the test has to have enough easy items to distinguish between people with different levels of low intelligence.
c. Incorrect See explanation for response b.
d. Incorrect See explanation for response b.

The correct answer is: confirm a diagnosis of Intellectual Disability.

How well did you know this?
1
Not at all
2
3
4
5
Perfectly
9
Q

A waiter is waiting on a group of professors from the local university. Being familiar with social loafing theory and desiring the largest tip possible, the waiter hopes that:
Select one:

A.
each professor will leave his/her own tip.

B.
the professors will contribute anonymously to a group tip.

C.
the professors will discuss how much tip to leave.

D.
one of the professors will volunteer to leave the tip for all of the professors.

A

Social loafing occurs when a person contributes less as group member than he/she would have contributed if acting alone.
a. CORRECT The waiter will get the most money if each person leaves his/her own tip since this will discourage social loafing.

b. Incorrect See explanation above.
c. Incorrect See explanation above.
d. Incorrect See explanation above.

The correct answer is: each professor will leave his/her own tip.

How well did you know this?
1
Not at all
2
3
4
5
Perfectly
10
Q

Several days after losing an important customer, David D., a 41-year-old sales representative, develops feelings of sadness and hopelessness. He tells his therapist that he has trouble even thinking about going out and “drumming up more business” and is concerned that his attitude may be having a negative effect on his relationships with other customers. David’s symptoms have lasted for nearly three weeks, and he has no history of a previous mental disorder. The most likely diagnosis for David is:
Select one:

A.
Adjustment Disorder with depressed mood.

B.
Adjustment Disorder, unspecified.

C.
Major Depressive Disorder.

D.
Acute Stress Disorder.

A

Answer A is correct: The nature and duration of David’s symptoms and the fact that they are apparently related to the loss of an important customer are consistent with the diagnostic criteria for Adjustment Disorder. Because his symptoms are those associated with depression (sadness and hopelessness), the correct diagnosis is Adjustment Disorder with depressed mood.

Answers B and D: See explanation for answer A.

Answer C: From the information provided in the question, it appears that David does not meet the full diagnostic criteria for Major Depressive Disorder.

The correct answer is: Adjustment Disorder with depressed mood.

How well did you know this?
1
Not at all
2
3
4
5
Perfectly
11
Q

When Angie plays the slot machines in Atlantic City, she will be reinforced on the basis of:
Select one:

A.
the number of responses between reinforcers with the number remaining the same from trial to trial.

B.
the number of responses between reinforcers with the number varying unpredictably from trial to trial.

C.
the interval of time between reinforcers with the length of time remaining constant from trial to trial.

D.
the interval of time between reinforcers with the length of time varying unpredictably from trial to trial.

A

Slot machines use intermittent reinforcement to reduce the likelihood of extinction.
a. Incorrect This describes a fixed ratio schedule.

b. CORRECT Angie will be reinforced on a variable ratio schedule, which is the intermittent schedule least vulnerable to extinction.
c. Incorrect This describes a fixed interval schedule.
d. Incorrect This describes a variable interval schedule.

The correct answer is: the number of responses between reinforcers with the number varying unpredictably from trial to trial.

How well did you know this?
1
Not at all
2
3
4
5
Perfectly
12
Q

Following a severe closed head injury caused by a car accident, a 26-year-old male exhibits anterograde and retrograde amnesia. Most likely, which of the following memories will return first during his recovery?
Select one:

A.
memory for the events that occurred just after the accident

B.
memory for events immediately before the accident

C.
memory for his birthday dinner six months prior to the accident

D.
memory for his high school graduation

A

Following moderate to severe head injury, a person is likely to show some degree of both anterograde and retrograde amnesia. In this situation, recovery of long-term memories typically involves a predictable pattern.
a. Incorrect Anterograde amnesia is usually more severe than retrograde amnesia, and the individual usually never recalls the incident that caused the amnesia or the events occurring after it because those events were never stored in long-term memory.

b. Incorrect See explanation for responses a and d.
c. Incorrect See explanation for responses a and d.
d. CORRECT Memories of more remote events ordinarily return first.

The correct answer is: memory for his high school graduation

How well did you know this?
1
Not at all
2
3
4
5
Perfectly
13
Q

In vivo exposure with response prevention has been found to be an effective treatment for Agoraphobia, Obsessive-Compulsive Disorder, and several other anxiety disorders. Studies investigating the effects of this treatment for this suggest that:
Select one:

A.
frequent, brief exposures are more effective than less frequent, longer exposures to the feared stimuli.

B.
people with high arousability are more responsive to in vivo treatments than people with low arousability.

C.
high anxiety provocation is not the key factor in its effectiveness.

D.
counterconditioning is a necessary component of treatment.

A

The goal of in vivo exposure is to extinguish the classically conditioned response by repeatedly exposing the person to the CS without the US.
a. Incorrect This is the opposite of what has been found to be true.

b. Incorrect Arousability has been linked to the effects of systematic desensitization, not in vivo exposure. Systematic desensitization is usually more successful for individuals low in arousability than for those high in arousability.
c. CORRECT As noted in the Learning Theory chapter of the written study materials, there is evidence that high anxiety arousal is NOT necessary for successful treatment with in vivo exposure.
d. Incorrect Counterconditioning is not a component of in vivo exposure.

The correct answer is: high anxiety provocation is not the key factor in its effectiveness.

How well did you know this?
1
Not at all
2
3
4
5
Perfectly
14
Q

Bandura’s social learning theory implies that, in organizational settings, a training program will be most effective when:
Select one:

A.
it provides immediate rewards for achieving goals.

B.
trainees participate in defining the program’s goals.

C.
trainees possess prerequisite skills before training begins.

D.
models are clearly and consistently reinforced for successful performance.

A

Bandura has provided guidelines for applying social learning theory to the workplace. See, e.g., R. Wood and A. Bandura, Social cognitive theory of organizational management, Academy of Management Review, 13, 361-384, 1999).
a. Incorrect See explanation for response c.

b. Incorrect See explanation for response c.
c. CORRECT Bandura stresses the importance of (1) self-efficacy beliefs; (2) intrinsic motivation; (3) focusing on overt activities and behaviors; and (4) ensuring that people have prerequisite skills.
d. Incorrect See explanation for response c. (Note that Bandura’s research indicated that it is not always necessary for the model to be rewarded in order for an observer to imitate the model’s behavior.)

The correct answer is: trainees possess prerequisite skills before training begins.

How well did you know this?
1
Not at all
2
3
4
5
Perfectly
15
Q

The treatment-of-choice for Specific Phobia is:
Select one:

A.
covert sensitization.

B.
exposure with response prevention.

C.
stimulus control.

D.
self-control therapy.

A

Answer B is correct: The treatment-of-choice for Specific Phobia is exposure with response prevention (especially in vivo exposure) that exposes the individual to the feared object or situation while preventing him or her from engaging in cognitive or behavioral avoidance.

Answers A, C, and D: See explanation for answer B.

The correct answer is: exposure with response prevention.

How well did you know this?
1
Not at all
2
3
4
5
Perfectly
16
Q

A psychology intern does not inform clients that he will be receiving supervision. This is:
Select one:

A.
ethical and a good practice since knowledge of the supervisory relationship could undermine his authority in therapy.

B.
ethical as long as he disguises the identity of the clients when discussing them with his supervisor.

C.
ethical as long as the client is aware that the psychologist is an intern.

D.
unethical.

A

To be in compliance with the APA’s Ethics Code, an intern must inform clients that he/she will be supervised and give the clients the name of the supervisor.
a. Incorrect See explanation for response d.

b. Incorrect See explanation for response d.
c. Incorrect See explanation for response d.
d. CORRECT Standard 10.01(c)states: “When the therapist is a trainee and the legal responsibility for the treatment provided resides with the supervisor, the client/patient, as part of the informed consent procedure, is informed that the therapist is in training and is being supervised and is given the name of the supervisor.”

The correct answer is: unethical.

How well did you know this?
1
Not at all
2
3
4
5
Perfectly
17
Q

As defined by Gestalt therapists, ________ is a boundary disturbance that involves passively accepting and incorporating elements of the enviroment without question.
Select one:

A.
retroflection

B.
confluence

C.
introjection

D.
deflection

A

Gestalt therapists identify several boundary disturbances which act as defenses that keep the individual from experiencing the present in a genuine and full way.
a. Incorrect Retroflection involves doing to someone else what one wants to do to others.

b. Incorrect Confluence refers to the absence of a boundary between the self and others.
c. CORRECT Introjection occurs when a person accepts facts, values, etc. from the environment without actually understanding or assimilating them.
d. Incorrect Deflection is the use of humor, abstract generalizations, and other distractions to avoid maintaining contact with the environment.

The correct answer is: introjection

How well did you know this?
1
Not at all
2
3
4
5
Perfectly
18
Q

When an examinee obtains a score of 110 on a test that has a standard error of measurement of 4, there is a 2 in 3 chance that his true score falls between:
Select one:

A.
108 and 112.

B.
106 and 114.

C.
104 and 116.

D.
102 and 118.

A

This question is asking about confidence intervals but does so in an unexpected way.
a. Incorrect See explanation for response b.

b. CORRECT To identify the correct answer to this question, you’d have to recognize that 2 in 3 is equivalent to a 2/3 chance, which is close to a 68% chance. A 68% confidence interval is calculated by adding and subtracting one standard error to and from the obtained score - which, in this case, gives you a confidence interval of 106 to 114.
c. Incorrect See explanation for response b.
d. Incorrect See explanation for response b.

The correct answer is: 106 and 114.

How well did you know this?
1
Not at all
2
3
4
5
Perfectly
19
Q

You conduct a research study to assess the effects of T.V. violence on aggressive behavior. The study will take place over three consecutive weeks. During the first week, you will observe 20 children during recess for five consecutive days and calculate their average number of aggressive acts. During the second week, you will have all children observe aggressive T.V. programs for two hours on three separate days. Then, during the third week, you will again observe the children during recess for five days and calculate their average number of aggressive acts. The biggest threat to the internal validity of this study is:
Select one:

A.
carryover effects.

B.
statistical regression.

C.
selection.

D.
history.

A

Answer D is correct: In any single-group design involving a before and after comparison, change in status on the DV might be due to the intervention or to an external event that occurred at about the same time the intervention was applied. In other words, the study’s internal validity is threatened by history.

Answer A: Carryover effects are a problem when two or more levels of an IV are applied to the same participants.

Answer B: Statistical regression is a problem when the group or groups have been selected because of their extreme status on the DV or a related variable, which is not the case in this study.

Answer C: Selection is a problem in between-groups studies when participants cannot be randomly assigned to the different groups. This is a within-subjects study, so selection is not a threat.

The correct answer is: history.

How well did you know this?
1
Not at all
2
3
4
5
Perfectly
20
Q

The criminal justice system provides intervention at what level of prevention?
Select one:

A.
primary

B.
secondary

C.
tertiary

D.
quartiary

A

Methods of prevention are classified based on the level of intervention. Primary prevention focuses on all members of an identified group. Secondary prevention targets specific individuals who are at high risk. Tertiary prevention intervenes after the incidence of the behavior.
a. Incorrect See explanation for response c.

b. Incorrect See explanation for response c.
c. CORRECT The criminal justice system intervenes after an individual engages in a criminal act.
d. Incorrect See explanation for response c.

The correct answer is: tertiary

How well did you know this?
1
Not at all
2
3
4
5
Perfectly
21
Q

Validity generalization studies make use of which of the following techniques?
Select one:

A.
multiple regression

B.
meta-analysis

C.
item response theory

D.
multitrait-multimethod matrix

A

Validity generalization studies are conducted to determine if validity coefficients are generalizable to situations and populations other than those included in any particular validity study. Schmidt and Hunter (1977), for example, conducted validity generalization studies to evaluate and compare the criterion-related validity of personnel selection procedures.
a. Incorrect Multiple regression uses multiple predictors to predict outcome on a single criterion measure.

b. CORRECT All forms of validity generalization involve the use of some form of meta-analysis to combine the results of multiple validity studies.
c. Incorrect Item response theory is an alternative to classical test theory for conceptualizing and constructing tests.
d. Incorrect The multitrait-multimethod matrix is used to evaluate a test’s construct validity.

The correct answer is: meta-analysis

How well did you know this?
1
Not at all
2
3
4
5
Perfectly
22
Q

As a treatment for Tourette’s Disorder, the selective serotonin reuptake inhibitors (SSRIs):
Select one:

A.
are more effective than haloperidol or pimozide for alleviating tics.

B.
are useful for reducing obsessive-compulsive symptoms that often accompany this disorder.

C.
cannot be used in conjunction with haloperidol or pimozide because of the risk for a toxicity syndrome.

D.
are contraindicated because of their exacerbating effects on tics.

A

Answer B is correct: A number of drugs are used to treat Tourette’s Disorder and comorbid symptoms. For example, an SSRI is usually useful for alleviating the obsessive-compulsive symptoms that often accompany this disorder.

Answers A, C, and D: See explanation for answer B.

The correct answer is: are useful for reducing obsessive-compulsive symptoms that often accompany this disorder.

How well did you know this?
1
Not at all
2
3
4
5
Perfectly
23
Q

A young man has just received a diagnosis of Narcissistic Personality Disorder. You can expect that he will display which of the following?
Select one:

A.
rapidly changing and shallow expression of emotions

B.
instability in self-image, interpersonal relationships, and mood

C.
grandiose behavior and a lack of empathy

D.
excessive emotionality and attention seeking

A

Answer C is correct: Narcissistic Personality Disorder involves a pervasive pattern of grandiosity, a need for admiration, and a lack of empathy.

Answer A: Rapidly changing and shallow expression of emotions are characteristic of Histroinic Personality Disorder.

Answer B: Instability in self-image, interpersonal relationships, and mood are characteristic of Borderline Personality Disorder.

Answer D: Excessive emotionality and attention seeking are characteristic of Histrionic Personality Disorder.

The correct answer is: grandiose behavior and a lack of empathy

How well did you know this?
1
Not at all
2
3
4
5
Perfectly
24
Q

Research on theory of mind has found that it is not until about ______ years of age that children understand that another person’s actions depend on that person’s beliefs (which may be false) rather than on the reality of the situation.
Select one:

A.
two to three

B.
four to five

C.
six to seven

D.
eight to nine

A

The research has found that theory of mind develops gradually during childhood in predictable stages.

a. Incorrect See explanation for answer b.
b. CORRECT By four to five years of age, children understand that a person’s beliefs about a situation may be false and that the person will act upon the false belief rather than the reality of the situation.
c. Incorrect See explanation for answer b.
d. Incorrect See explanation for answer b.

The correct answer is: four to five

How well did you know this?
1
Not at all
2
3
4
5
Perfectly
25
Q

Universal symbols in the delusions and hallucinations of psychotic patients would be of most interest to:
Select one:

A.
Perls.

B.
Sullivan.

C.
Adler.

D.
Jung.

A

The term “universal symbols” should have helped you recognize the correct answer to this question.
a. Incorrect See explanation for response d.

b. Incorrect See explanation for response d.
c. Incorrect See explanation for response d.
d. CORRECT Of the individuals listed, only Jung incorporates the notion of universal symbols into his work. When working with psychotic patients, Jung noticed a great deal of similarity in the content of their delusions and hallucinations, and this led to the development of his theory of archetypes and the collective unconscious.

The correct answer is: Jung.

How well did you know this?
1
Not at all
2
3
4
5
Perfectly
26
Q

A non-custodial parent asks Dr. Maxine Miller, a school psychologist, for the results of the tests she recently administered to his 5th grade son. If Dr. Miller complies with the father’s request, she will have acted:
Select one:

A.
legally and ethically.

B.
legally but unethically.

C.
illegally and unethically.

D.
illegally but ethically.

A

The rights of non-custodial parents may be limited in terms of obtaining psychological services for their children. However, custody status does not necessarily affect access to certain kinds of information.
a. CORRECT The laws relevant to this situation vary from jurisdiction to jurisdiction but, in general, non-custodial parents have the right to obtain information about their children. In California, for example, the law states that “Notwithstanding any other provision of law, access to records and information pertaining to a minor child, including but not limited to medical, dental, and school records, shall not be denied to a parent because such parent is not the child”s custodial parent” (Civil Code 4600.5).

b. Incorrect See explanation for response a.
c. Incorrect See explanation for response a.
d. Incorrect See explanation for response a.

The correct answer is: legally and ethically.

How well did you know this?
1
Not at all
2
3
4
5
Perfectly
27
Q

Which of the following memory techniques relies on the use of visual images?
Select one:

A.
acronym

B.
chunking

C.
acrostic

D.
keyword method

A

For the exam, you want to be familiar with the terms listed in the responses so that you can identify the correct answer to questions like this one.
a. Incorrect An acronym involves constructing a word from the first letter of the words to be memorized.

b. Incorrect Chunking involves grouping items that are to be recalled and is useful for increasing the amount of information that can be held in short-term memory.
c. Incorrect An acrostic is a phrase or rhyme that is composed of words that begin with the first letter of the words to be memorized.
d. CORRECT The keyword method is useful for paired associate tasks in which two words must be linked (e.g., foreign-language learning). It involves constructing a visual image that combines the two words.

The correct answer is: keyword method

How well did you know this?
1
Not at all
2
3
4
5
Perfectly
28
Q

At age 8, a girl receives a WISC FSIQ score of 144. When she is retested at age 12, she obtains a score of 138. Which of the following best accounts for this decline in her IQ score?
Select one:

A.
demand characteristics

B.
cohort effects

C.
normal aging effects

D.
statistical regression

A

Statistical regression refers to the tendency of extreme scores to regress toward the mean on retesting and may threaten a study’s internal validity whenever participants are chosen on the basis of their extreme scores on a pretest.
a. Incorrect See explanation for response d.

b. Incorrect See explanation for response d.
c. Incorrect See explanation for response d.
d. CORRECT Although statistical regression is typically used in the context of scores obtained by a group of examinees (rather than an individual examinee), of the answers given, it is the best choice.

The correct answer is: statistical regression

How well did you know this?
1
Not at all
2
3
4
5
Perfectly
29
Q

In infants, auditory localization is first apparent:
Select one:

A.
shortly after birth.

B.
between 3 and 4 months of age.

C.
between 6 and 7 months of age.

D.
between 8 and 9 months of age.

A

Auditory localization is the ability to orient toward the direction of a sound.
a. CORRECT Some auditory localization is evident soon after birth, but this ability seems to disappear between two and four months of age and then reappears and gradually improves during the first year of life.

b. Incorrect See explanation for response a.
c. Incorrect See explanation for response a.
d. Incorrect See explanation for response a.

The correct answer is: shortly after birth.

How well did you know this?
1
Not at all
2
3
4
5
Perfectly
30
Q

Raymond R. tells Dr. Terry Fide that his partner is talking about harming Dr. Fide because he’s not happy with the advice that Raymond is getting in therapy. Dr. Fide knows that the partner has engaged in violent behavior in the past. In this situation, Dr. Fide:
Select one:

A.
should continue seeing Raymond in therapy and ask that Raymond bring his partner to the next session to discuss the issue.

B.
should continue seeing Raymond in therapy but, with his permission, make a report to the police department.

C.
may terminate therapy with Raymond but only after providing him with pretermination counseling and appropriate referrals.

D.
may terminate therapy with Raymond.

A

This issue is addressed in Standard 10.10(b) of the APA’s Ethics Code and the Values Statement for Principle II of the Canadian Code of Ethics for Psychologists.
a. Incorrect See explanation for response d.

b. Incorrect See explanation for response d.
c. Incorrect See explanation for response d.
d. CORRECT Psychologists may terminate therapy when their well-being is threatened by the client or a person the client knows. In this situation, it is not necessary to provide notification of termination, pretermination counseling, or referrals. See, e.g., C. B. Fisher, Decoding the Ethics Code: A practical guide for psychologists, Thousand Oaks, CA, 2003.

The correct answer is: may terminate therapy with Raymond.

How well did you know this?
1
Not at all
2
3
4
5
Perfectly
31
Q

Six weeks after terminating therapy, Wilma W. still owes you for the last four therapy sessions. You have sent Wilma a letter asking that she contact you about her outstanding fees, but she has not responded and you would like to use a collection agency to obtain the money she owes you. As an ethical psychologist:
Select one:

A.
you will not contact a collection agency unless you informed Wilma that this was your policy during the original informed consent process.

B.
you will contact a collection agency only if Wilma contacts you and informs you that she doesn’t intend to pay her bill.

C.
you will contact a collection agency only after notifying Wilma of your intent to do so if she doesn’t respond by a specific date.

D.
you will not use a collection agency under any circumstances.

A

This issue is explicitly addressed in Standard 6.04(e) of the APA’s Ethics Code and indirectly addressed in Principle I.12 of the Canadian Code of Ethics.
a. Incorrect See explanation for response c.

b. Incorrect See explanation for response c.
c. CORRECT Standard 6.04(e) states: “If the recipient of services does not pay for services as agreed, and if psychologists intend to use collection agencies or legal measures to collect the fees, psychologists first inform the person that such measures will be taken and provide that person an opportunity to make prompt payment.”
d. Incorrect See explanation for response c.

The correct answer is: you will contact a collection agency only after notifying Wilma of your intent to do so if she doesn’t respond by a specific date.

How well did you know this?
1
Not at all
2
3
4
5
Perfectly
32
Q

Studies investigating the impact of viewing violent pornography have generally supported the predictions of which of the following?
Select one:

A.
frustration-aggression hypothesis

B.
catharsis hypothesis

C.
social learning theory

D.
vicious circle hypothesis

A

The research has found that exposure to pornography with violent themes increases the viewer’s aggressive behaviors as well as acceptance of violence against women.
a. Incorrect See explanation for response c.

b. Incorrect See explanation for response c.
c. CORRECT Social learning theory predicts that we acquire behaviors by observing others engage in those behaviors. Therefore, its predictions are consistent with the results of studies investigating the effects of viewing violent pornography.
d. Incorrect See explanation for response c.

The correct answer is: social learning theory

How well did you know this?
1
Not at all
2
3
4
5
Perfectly
33
Q

A researcher designs a study in which participants will be hospitalized patients who have legal guardians. The hospital research committee has approved the project, which involves several treatments, some of which include experimental drugs that may cause harmful side effects. Before beginning the study, the researcher should:
Select one:

A.
obtain informed consents from the patients if possible.

B.
obtain informed consents from the patients’ guardians.

C.
obtain informed consents from the patients’ guardians and assent from the patients.

D.
none of the above are necessary since the study has been approved by the hospital’s research committee.

A

To be consistent with ethical guidelines, when an individual is not competent to give informed consent, a consent must be obtained from his/her legal guardian. In addition, the individual should be provided with information about the study in language he/she can understand.
a. Incorrect This is appropriate but insufficient.

b. Incorrect This is appropriate but insufficient.
c. CORRECT This is most consistent with ethical guidelines (see Standard 3.10 of the APA’s Ethics Code).
d. Incorrect This is not true and would represent a breach of the ethical guidelines.

The correct answer is: obtain informed consents from the patients’ guardians and assent from the patients.

How well did you know this?
1
Not at all
2
3
4
5
Perfectly
34
Q

Damage to the postcentral gyrus is most likely to impair which of the following?
Select one:

A.
sense of touch

B.
motor coordination

C.
vision

D.
executive cognitive functions

A

The postcentral gyrus contains the primary somatosensory cortex.
a. CORRECT The primary somatosensory cortex mediates touch and other skin and muscle sensations for the contralateral (opposite) side of the body.

b. Incorrect See explanation above.
c. Incorrect See explanation above.
d. Incorrect See explanation above.

The correct answer is: sense of touch

How well did you know this?
1
Not at all
2
3
4
5
Perfectly
35
Q

As defined by Salvador Minuchin, triangulation is:
Select one:

A.
a therapeutic technique that helps family members achieve insight.

B.
a structural configuration that is desirable because it improves communication.

C.
a chronic boundary problem that serves a conflict-diffusing function.

D.
a chronic boundary disturbance involving “talking through” rather than “talking to.”

A

Although the term “triangulation” is probably most associated with Bowen, Minuchin and other family therapists also use this term.
a. Incorrect See explanation for response c.

b. Incorrect See explanation for response c.
c. CORRECT Even if you are unfamiliar with Minuchin’s use of this term, knowing that he focuses on boundary disturbances and emphasizes the role of stress in family dysfunction would have helped you pick this as the correct response. As noted in the Clinical Psychology chapter of the written study materials, Minuchin describes several types of rigid triads (triangles) that all serve to reduce conflict and stress - i.e., triangulation, detouring, and stable coalition.
d. Incorrect See explanation for response c.

The correct answer is: a chronic boundary problem that serves a conflict-diffusing function.

How well did you know this?
1
Not at all
2
3
4
5
Perfectly
36
Q

During the process of developing a training program, you evaluate its effects on trainee satisfaction and learning to determine what changes to the program are needed to achieve maximal effectiveness. This is an example of:
Select one:

A.
needs assessment.

B.
functional analysis.

C.
summative evaluation.

D.
formative evaluation.

A

This question is a little vague, but it appears that the purpose of the evaluation is to make changes in the training program while it is being developed.
a. Incorrect A needs assessment is conducted to determine training needs.

b. Incorrect A functional analysis is used by behavioral psychologists to obtain information about a target behavior.
c. Incorrect A summative evaluation is conducted to determine the outcomes of a program and is less geared toward guiding future changes than a formative evaluation is.
d. CORRECT Although the question isn’t entirely clear, the evaluation sounds more like a formative one because it is conducted during program development and its results will be used to make necessary changes in the program.

The correct answer is: formative evaluation.

How well did you know this?
1
Not at all
2
3
4
5
Perfectly
37
Q

Research comparing the personality characteristics and psychiatric symptoms of women reporting repressed versus continuous memories of childhood sexual abuse has found that:
Select one:

A.
the two groups of women do not differ significantly in terms of personality characteristics or psychiatric symptoms.

B.
women reporting repressed memories differ from those with continuous memories in terms of dissociation but not with regard to other characteristics or symptoms.

C.
women reporting continuous memories exhibit more symptoms of PTSD than those reporting repressed memories but the two groups do not differ with regard to other characteristics or symptoms.

D.
the two groups of women show significant differences with regard to both personality characteristics and psychiatric symptoms.

A

This issue was addressed by R. J. McNally et al. [Personality profiles, dissociation, and absorption in women reporting repressed, recovered, or continuous memories of childhood sexual abuse, Journal of Consulting and Clinical Psychology, 68(6), 1033-1037, 2000]. Individuals in this study reporting repressed memories said they believed they had been sexually abused as children but had no autobiographical memories of the abuse.
a. Incorrect See explanation for response d.

b. Incorrect See explanation for response d.
c. Incorrect See explanation for response d.
d. CORRECT McNally et al. found that women with repressed memories scored higher on measures of absorption and dissociation and reported more symptoms of depression and PTSD than did women with continuous memories of childhood abuse. (Women with continuous memories did not differ significantly from nonabused women on these measures.)

The correct answer is: the two groups of women show significant differences with regard to both personality characteristics and psychiatric symptoms.

How well did you know this?
1
Not at all
2
3
4
5
Perfectly
38
Q

During her second therapy session, a client tells Dr. O. Pine that she is pregnant, doesn’t want to have the baby, and is planning on having an abortion. Dr. Pine is very much against abortions and is concerned that his feelings might interfere with the therapeutic relationship. As an ethical psychologist, Dr. Pine should:
Select one:

A.
discuss this issue with the client and allow her to decide if she wants to continue seeing him.

B.
continue to see the woman but make sure his feelings don’t adversely impact treatment.

C.
seek consultation with a collegue but continue to see the woman while doing so.

D.
refer the woman to another therapist.

A

Psychologists are not required to see every client who seeks their services.
a. Incorrect See explanation for response d.

b. Incorrect See explanation for response d.
c. Incorrect See explanation for response d.
d. CORRECT It’s appropriate not to treat clients when your beliefs and attitudes may impact your objectivity or otherwise harm the therapeutic process. The best course of action is this situation is to make a referral.

The correct answer is: refer the woman to another therapist.

How well did you know this?
1
Not at all
2
3
4
5
Perfectly
39
Q

Lithium is often the treatment-of-choice for mania, with the research indicating that 80% of patients have a favorable response to it. In recent years, however, there has been increasing interest in prescribing __________, which seem to be as effective as lithium for controlling acute manic states and stabilizing mood.
Select one:

A.
SSRIs

B.
antipsychotic drugs

C.
benzodiazepines

D.
anticonvulsant medications

A

Although lithium has been the standard treatment for Bipolar Disorder to reduce mania and mood fluctuations, alternative drugs have been found to be similarly effective.
a. Incorrect See explanation for response d.

b. Incorrect See explanation for response d.
c. Incorrect See explanation for response d.
d. CORRECT Of the drugs listed, only anticonvulsant drugs (especially carbamazapine and valproate) have been found to be comparable to lithium in their beneficial effects.

The correct answer is: anticonvulsant medications

How well did you know this?
1
Not at all
2
3
4
5
Perfectly
40
Q

After a training program has been administered to employees, an organizational psychologist wants to determine if the instruction met its objectives in terms of the employees’ on-the-job performance. What type of evaluation will he conduct?
Select one:

A.
summative

B.
formative

C.
primary

D.
secondary

A

Program evaluations are usually conducted both while the program is being developed and after the program has been implemented.
a. CORRECT A summative evaluation is conducted after the administration of a program to determine if the program’s goals have been achieved.

b. Incorrect Formative evaluations are conducted during the development of a program to determine whether the program should be altered to improve its effectiveness.
c. Incorrect This is a distractor term.
d. Incorrect This is a distractor term.

The correct answer is: summative

How well did you know this?
1
Not at all
2
3
4
5
Perfectly
41
Q

Krumboltz’s (1996) theory of career decision-making emphasizes which of the following?
Select one:

A.
the match between the individual’s ego development and his/her career development

B.
the match between the individual’s characteristics and the characteristics of the job

C.
the individual’s ability to acquire skills that allow him/her to respond to changes in work requirements

D.
the degree to which a job fosters the individual’s ability to achieve career maturity

A
How well did you know this?
1
Not at all
2
3
4
5
Perfectly
42
Q

You receive a call from an emergency room psychiatrist who is currently treating a former client of yours who has just attempted suicide. The client has given consent for you to provide the psychiatrist with information from her records but she currently owes you for the last three sessions of therapy. In this situation:
Select one:

A.
you have no obligation to provide the records to the psychiatrist.

B.
you may withhold the records if the client has not responded to your requests for payment.

C.
you may withhold the records as long as the client was advised of this policy during the informed consent process.

D.
you may not withhold the records.

A

Answer D is correct. This issue is directly addressed in Standard 6.03 of the APA’s Ethics Code and indirectly addressed in Principles II.1 and II.2 of the Canadian Code of Ethics for Psychologists. Although psychologists have a right to be paid for their services, they must also avoid harming their clients, and withholding records in an emergency situation due to nonpayment of fees would jeopardize the welfare of the client.

The correct answer is: you may not withhold the records.

How well did you know this?
1
Not at all
2
3
4
5
Perfectly
43
Q

Patients with traumatic brain injury are likely to obtain the lowest score on which of the following WAIS-IV Indexes?
Select one:

A.
Verbal Comprehension

B.
Perceptual Reasoning

C.
Working Memory

D.
Processing Speed

A

Answer D is correct: The impact of brain injury on WAIS-IV scores is reported in the test’s Technical and Interpretive Manual, which indicates that individuals in the standardization sample with traumatic brain injury scored highest on Verbal Comprehension followed by Perceptual Reasoning, Working Memory, and Processing Speed.

The correct answer is: Processing Speed

How well did you know this?
1
Not at all
2
3
4
5
Perfectly
44
Q

Spontaneous recovery is associated with which of the following?
Select one:

A.
higher-order conditioning

B.
stimulus discrimination

C.
extinction

D.
overshadowing

A

Spontaneous recovery occurs when an extinguished conditioned response recurs without additional conditioning trials.
a. Incorrect See explanation for response c.

b. Incorrect See explanation for response c.
c. CORRECT Pavlov found that, following extinction trials, dogs in his studies often spontaneously emitted the condtioned reponse, and he referred to this as spontaneous recovery. Spontaneous recovery is the reason why it is often necessary to conduct multiple extinction sessions in order to eliminate a conditioned response.
d. Incorrect See explanation for resopnse c.

The correct answer is: extinction

How well did you know this?
1
Not at all
2
3
4
5
Perfectly
45
Q

Which of the following is LEAST true about practitioners of feminist therapy?
Select one:

A.
They encourage female clients to express their anger.

B.
They avoid traditional diagnostic labels.

C.
They encourage female clients to accept personal responsibility for their victimization.

D.
They promote an egalitarian therapeutic relationship through self-disclosure.

A

A key distinguishing feature of feminist therapy is its assumption that behavior must be interpreted within an oppressive social context.
a. Incorrect Feminist therapists do encourage women to express any anger they may have repressed and to accept the consequences of expressing that anger.

b. Incorrect Feminist therapists avoid traditional labels whenever possible, including diagnostic labels.
c. CORRECT Although feminist therapists want women to take control of their lives, they avoid revictimization by not placing the blame for problems on the clients themselves.
d. Incorrect An egalitarian relationship is a crucial aspect of feminist therapy. Feminist therapists use several techniques to foster an egalitarian relationship including appropriate self-disclosure.

The correct answer is: They encourage female clients to accept personal responsibility for their victimization.

How well did you know this?
1
Not at all
2
3
4
5
Perfectly
46
Q

Investigators have found that, after engaging in strenuous physical activity, research participants tend to get angrier when provoked by an insult and tend to be more romantically or sexually attracted when they meet a potential partner. These findings provide support for which of the following?
Select one:

A.
misattribution hypothesis

B.
confirmatory bias

C.
catharsis hypothesis

D.
psychological reactance

A

This question describes situations in which prior physiological arousal affects subsequent reactions. Of the theories listed, only one applies to this type of situation.
a. CORRECT You probably had trouble answering this question if you did not know that Schachter’s two-factor theory is also referred to as the misattribution hypothesis because it predicts that arousal produced by one source (e.g., strenuous physical activity) can be misattributed to another source (e.g., an insult or a potential romantic or sexual partner). In other words, the arousal gets “transferred” to another source. On the exam, you may encounter a few questions that use terms you are unfamiliar with; and, for those question, the process of elimination might help you identify the correct answer.

b. Incorrect The confirmatory bias (which is also known as the confirmation bias) predicts that people tend to seek or pay attention to information that confirms their current beliefs.
c. Incorrect The catharsis hypothesis predicts that an act of aggression reduces a person’s arousal level which, in turn, decreases the likelihood that he/she will act aggressively in the near future.
d. Incorrect Psychological reactance is the tendency of a person to resist being influenced by others (usually by doing the opposite of what is desired or expected) when the person believes that his/her personal freedom is being threatened.

The correct answer is: misattribution hypothesis

How well did you know this?
1
Not at all
2
3
4
5
Perfectly
47
Q

A diagnosis of Reactive Attachment Disorder cannot be assigned without the presence of which of the following?
Select one:

A.
evidence of extreme insufficient care

B.
developmentally inappropriate social relatedness that began prior to age 2

C.
a pattern of inappropriate social relatedness in at least two settings

D.
the presence of one or more developmental delays

A

Answer A is correct: Reactive Attachment Disorder is characterized by inappropriate social relatedness that is related to extreme insufficient (pathogenic) care.

Answer B: The diagnosis requires that symptoms begin prior to age 5 (not age 2).

Answer C: The diagnosis does not require signs of inappropriate social relatedness in at least two settings.

Answer D: Developmental delays are not required for the diagnosis.

The correct answer is: evidence of extreme insufficient care

How well did you know this?
1
Not at all
2
3
4
5
Perfectly
48
Q

Outward signs of the self-conscious emotions of jealousy, empathy, and embarassment are first evident between ________ of age.
Select one:

A.
6 to 10 months

B.
10 to 18 months

C.
18 to 24 months

D.
24 to 30 months

A

It is not until the second year of life that children begin to exhibit outward signs of self-conscious emotions.
a. Incorrect See explanation for response c.

b. Incorrect See explanation for response c.
c. CORRECT Jealousy, empathy, and embarassment are among the first self-conscious emotions exhibited by young children.
d. Incorrect See explanation for response c.

The correct answer is: 18 to 24 months

How well did you know this?
1
Not at all
2
3
4
5
Perfectly
49
Q

During the first session of a therapy group, the therapist emphasizes the importance of self-disclosure. Although many of the members do not go along with this group norm initially, after several weeks, most of the members are beginning to self-disclose. According to Irvin Yalom, the willingness to self-disclose during this stage of group development is most likely attributable to which of the following?
Select one:

A.
increased role conformance

B.
reduced defensiveness

C.
increased group cohesiveness

D.
reduced role induction

A

Yalom (1985) proposed that, regardless of the type of therapy group or the theoretical background of the group leader, self-disclosure by participants is always an essential group norm.
a. Incorrect See explanation for response c.

b. Incorrect See explanation for response c.
c. CORRECT Yalom describes several initial stages that new groups typically pass through. By the third stage, as the result of increasing group cohesiveness, group members begin to adhere to the norm of self-disclosure even if they were resistant to doing so to begin with.
d. Incorrect See explanation for response c.

The correct answer is: increased group cohesiveness

How well did you know this?
1
Not at all
2
3
4
5
Perfectly
50
Q

A researcher uses the chi-square test to determine if the ethnic/racial make-up (i.e., White, African-American, Latino/Hispanic, Asian, Other) of a particular community on the West Coast differs significantly from a similar-sized community on the East Coast. Ten years later, she returns to the West Coast community and finds that it has not changed dramatically in size but has changed in terms of racial and ethnic group membership. The researcher now wants to determine if the West Coast community’s current ethnic/racial make-up differs significantly from the make-up she observed in that community ten years earlier and plans to compare the number of individuals in each ethnic/racial category for the two years (2000 and 2010). The researcher cannot use a chi-square test to analyze the data for her current study because:
Select one:

A.
the study now includes two independent variables.

B.
the data violate the assumption of independence.

C.
the data violate the assumption of random assignment.

D.
all of the above.

A

The use of any statistical test requires not only that the data are measured on a particular scale but also that certain assumptions be met.
a. Incorrect A multiple-sample chi-square test can be used when a study includes two or more independent variables.

b. CORRECT An assumption for the chi-square test is that observations are independent, which means that each individual can appear in only one category. In this study, people who lived in the community in both 2000 and 2010 would appear in two categories, which would violate this assumption.
c. Incorrect Random assignment is not an assumption for the chi-square test.
d. Incorrect Only response b is correct.

The correct answer is: the data violate the assumption of independence.

How well did you know this?
1
Not at all
2
3
4
5
Perfectly
51
Q

Dr. Smith is renting office space from Dr. Jones and providing him with secretarial services. Dr. Jones charges Dr. Smith on a “per patient” basis. Dr. Jones will be receiving occasional referrals from Dr. Smith. This arrangement is:
Select one:

A.
unethical because it represents “fee-splitting.”

B.
unethical because it represents a “multiple relationship.”

C.
ethical as long as the per patient fee is based on Dr. Jones’s actual costs.

D.
ethical as long as the per patient fee does not include a referral fee.

A

The correct answer is C. Although “fee-splitting” is not explicitly prohibited by the Ethics Code, certain restrictions apply when client fees will be shared. The per-patient fee must reflect Dr. Smith’s actual costs and can include any costs involved in making a referral.

Answer D is incorrect: A referral fee is acceptable as long as it represents the costs involved in making the referral (e.g., the time Dr. Smith spends preparing a client’s file or discussing the case with Dr. Jones).

The correct answer is: ethical as long as the per patient fee is based on Dr. Jones’s actual costs.

How well did you know this?
1
Not at all
2
3
4
5
Perfectly
52
Q

A criterion has a mean of 100 and a standard deviation of 9. Based on this information, you can conclude that the criterion’s standard error of estimate is between:
Select one:

A.
9 and 100.

B.
0 and 81.

C.
0 and 9.

D.
0 and 3.

A

Knowing the formula for the standard error of estimate may have helped you identify the correct answer to this question: The standard error of estimate is calculated by multiplying the standard deviation of the criterion scores times the square root of 1 minus the validity coefficient squared.
a. Incorrect See explanation for response c.

b. Incorrect See explanation for response c.
c. CORRECT The standard error of estimate equals 0 when the validity coefficient is equal to 1 (its maximum value) and equals the standard deviation when the validity coefficient is equal to 0 (its minimum value). Therefore, the standard error ranges in value from 0 to the size of the standard deviation (which, in this case, is 9).
d. Incorrect See explanation for response c.

The correct answer is: 0 and 9.

How well did you know this?
1
Not at all
2
3
4
5
Perfectly
53
Q

Cluster analysis would be most useful for:
Select one:

A.
developing a classification scheme for individuals with various combinations of depressive symptoms.

B.
testing causal hypotheses about the factors that underlie the development of depression.

C.
identifying the optimal combination of tests to use to predict an individuals risk for developing major depressive disorder.

D.
systematically recording behavioral data while observing individuals with depressive symptoms.

A

As its name suggests, cluster analysis is used to “cluster” or categorize individuals. Cluster analysis can be either exploratory or confirmatory. In the first case, the goal is to identify clusters of people or other variables; in the latter, the goal is to confirm a pre-existing classification scheme.
a. CORRECT Of the situations described in the answers, this one is best suited for cluster analysis, which is useful for confirming or developing a classification scheme.

b. Incorrect Cluster analysis is not used to test causal hypotheses.
c. Incorrect Cluster analysis would not be useful for this purpose.
d. Incorrect Cluster analysis is a type of statistical analysis and is not used to record behavioral observations.

The correct answer is: developing a classification scheme for individuals with various combinations of depressive symptoms.

How well did you know this?
1
Not at all
2
3
4
5
Perfectly
54
Q

According to the learned helplessness model, children who consistently have extreme negative emotional reactions to failure in school are most likely attributing their academic problems to:
Select one:

A.
lack of ability.

B.
lack of effort.

C.
bad luck.

D.
task difficulty.

A

The negative reactions of the children described in this question are characteristic of learned helplessness.
a. CORRECT The learned helplessness model predicts that, when people view their failures as the result of global, stable, and internal causes (e.g., lack of ability), they are likely to respond to failure with depression or other strong negative emotion.

b. Incorrect See explanation for response a.
c. Incorrect See explanation for response a.
d. Incorrect See explanation for response a.

The correct answer is: lack of ability.

How well did you know this?
1
Not at all
2
3
4
5
Perfectly
55
Q

“Multicollinearity” could be expected to:
Select one:

A.
cause the values of the regression coefficients to vary substantially from sample to sample.

B.
cause the multiple correlation coefficient to vary substantially from sample to sample.

C.
increase as the number of dependent variables decreases.

D.
decrease as the correlation between independent variables increases.

A

Multicollinearity occurs when predictors in a multiple regression equation correlate highly with one another.
a. CORRECT When predictors are correlated and the predictors and criterion are administered to different samples and new regression coefficients (beta weights) are derived for each sample, there tends to be a great deal of fluctuation in the size of the weights. (It isn’t necessary for you to understand why this is so for the exam – just know that it is!)

b. Incorrect The actual relationship between the predictors and criterion shouldn’t change much from sample to sample as the result of multicollinearity, so the multiple correlation coefficient won’t change substantially.
c. Incorrect You may have been able to eliminate this response because it doesn’t really make any sense.
d. Incorrect By definition, multicollinearity INCREASES as the correlation between predictors increases.

The correct answer is: cause the values of the regression coefficients to vary substantially from sample to sample.

How well did you know this?
1
Not at all
2
3
4
5
Perfectly
56
Q

An assumption underlying item response theory is that the standard error of measurement:
Select one:

A.
is larger for shorter tests than for longer tests.

B.
is larger for examinees with average ability.

C.
is larger for examinees with very high or very low ability.

D.
is not affected by the test length or the ability level of the examinees.

A

Item response theory (IRT) differs from classical test theory in terms of several assumptions.
a. Incorrect One difference between IRT and classical test theory is that IRT assumes that, when certain conditions are met, shorter tests can be more reliable than longer tests.

b. Incorrect See explanation for response c.
c. CORRECT Classical test theory is based on the assumption that the standard error of measurement is a constant that applies to all examinees, regardless of their level of ability. In contrast, IRT is based on the assumption that the standard error of measurement increases at the extremes of a distribution – i.e., is larger for individuals with very high or low ability than for those with average ability. See, e.g., R. J. Gregory, Psychological testing: History, principles, and applications, Boston, Pearson Education Group, 2004.
d. Incorrect See explanation for response c.

The correct answer is: is larger for examinees with very high or very low ability.

How well did you know this?
1
Not at all
2
3
4
5
Perfectly
57
Q

You are most likely to report “feeling crowded” in a high density situation if you are watching a:
Select one:

A.
humorous film.

B.
sexual film.

C.
violent film.

D.
documentary film.

A

This question is referring to a specific study on the effects of crowding (S. Worchel and E. H. Brown, The role of plausibility in influencing environmental attributions, Journal of Experimental Social Psychology, 20, 86-96, 1984).
a. Incorrect See explanation for response d.

b. Incorrect See explanation for response d.
c. Incorrect See explanation for response d.
d. CORRECT Participants in this study reported feeling more crowded while watching a nonarousing (documentary) film than while watching an arousing (humorous, sexual, or violent) film. Moreover, they said they enjoyed the arousing film more under crowded conditions.

The correct answer is: documentary film.

How well did you know this?
1
Not at all
2
3
4
5
Perfectly
58
Q

A “cover story” that causes research participants to think that the purpose of the study they are participating in is something other than what it really is would be most helpful for controlling which of the following?
Select one:

A.
Hawthorne effect

B.
Rosenthal effect

C.
demand characteristics

D.
differential attrition

A

In the situation described in this question, participants will not know the real purpose of the study. As long as you’re familiar with the phenomena listed in the answers, you should have been able to recognize which phenomenon would be impacted by this lack of knowledge.
a. Incorrect The Hawthorne effect is the tendency for the performance of research participants to improve as the result of the attention they receive as research participants. In other words, it is knowing that they are in a study (not knowing the purpose of the study) that impacts performance.

b. Incorrect The Rosenthal (self-fulfilling prophecy) effect refers to the impact of a teacher”s expectations about the performance of his/her students on their actual performance.
c. CORRECT Demand characteristics are cues in the experimental setting that inadvertently convey information to participants about what behavior is expected. If participants do not know what the purpose of the study is, they will be less susceptible to the effects of any cues that happen to be present.
d. Incorrect Differential attrition occurs when participants who drop out of one group differ in a systematic way from those who drop out of another group and this difference affects the study’s results.

The correct answer is: demand characteristics

How well did you know this?
1
Not at all
2
3
4
5
Perfectly
59
Q

If you want to determine the degree of association between variable X and variable Y when the effects of a third variable (Z) have been statistically removed from both X and Y, you would use:
Select one:

A.
zero-order correlation.

B.
second-order correlation.

C.
multiple correlation.

D.
partial correlation.

A

You may have been able to identify the correct answer to this question even if you have never heard of partial correlation since it sounds like what it is.
a. Incorrect A zero-order correlation is the correlation between two variables.

b. Incorrect In the context of correlation, “order” refers to the number of variables that are being controlled for. A first-order correlation controls for one other variable (which is what is described in this question), while a second-order correlation controls for two other variables.
c. Incorrect A multiple correlation coefficient is the correlation between three or more variables.
d. CORRECT Partial correlation, a type of first-order correlation, is used to determine the degree of association between two variables when the effects of a third variable have been removed from both X and Y.

The correct answer is: partial correlation.

How well did you know this?
1
Not at all
2
3
4
5
Perfectly
60
Q

Use of multicomponent behavior therapy in conjunction with nicotine replacement therapy as an intervention for cigarette smoking:
Select one:

A.
is no better than nicotine replacement therapy alone.

B.
is no better than behavior therapy alone.

C.
is better than nicotine replacement therapy alone only for individuals who have developed tolerance for nicotine.

D.
is better than nicotine replacement therapy alone, especially in terms of long-term effects.

A

Answer D is correct: There is some evidence that, in terms of short-term effects, a combined treatment does not have significant advantages over nicotine replacement therapy alone. However, in terms of long-term abstinence, the combined treatment is best. See R. C. Klesges, K. D. Ward, and M. DeBon, Smoking cessation: A successful behavioral/pharmacologic interface, Clinical Psychology Review, 16(6), 479-496, 1996.

Answers A, B, and C: See explanation for answer D.

The correct answer is: is better than nicotine replacement therapy alone, especially in terms of long-term effects.

How well did you know this?
1
Not at all
2
3
4
5
Perfectly
61
Q

Research comparing outcomes for children who are actively rejected or neglected by their peers has found that:
Select one:

A.
rejected children are better adjusted than neglected children and are more likely to experience a change in social status when they change social groups.

B.
rejected and neglected children are similar in terms of adjustment although neglected children are less likely to experience a change in social status when they change social groups.

C.
rejected and neglected children are similar in terms of adjustment although neglected children are more likely to experience a change in social status when they change social groups.

D.
neglected children are better adjusted than rejected children and are more likely to experience a change in social status when they change social groups.

A

In terms of peer acceptance, children may be classified as popular, rejected, or neglected.
a. Incorrect See explanation for response d.

b. Incorrect See explanation for response d.
c. Incorrect See explanation for response d.
d. CORRECT As noted in the Lifespan Development chapter of the written study materials, the research has found that, overall, the outcomes for neglected children are better than those for rejected children. In addition, neglected children are more likely to experience a change in peer status when they change schools or social groups.

The correct answer is: neglected children are better adjusted than rejected children and are more likely to experience a change in social status when they change social groups.

How well did you know this?
1
Not at all
2
3
4
5
Perfectly
62
Q

With regard to ethical guidelines, a sexual relationship between a psychologist and an adult student:
Select one:

A.
is prohibited under any conditions.

B.
is prohibited only when the relationship is likely to impair the psychologist’s effectiveness as an instructor.

C.
is prohibited only when the psychologist currently has evaluative authority over the student.

D.
is prohibited when the psychologist currently has or is likely to have evaluative authority over the student.

A

For the exam, you want to be familiar with ethical guidelines for sexual relationships with students, supervisees, and clients. These guidelines are discussed in the chapter on ethics and professional issues in the written study materials.
a. Incorrect See explanation for response d.

b. Incorrect See explanation for response d.
c. Incorrect See explanation for response d.
d. CORRECT Sexual relationships with students and supervisees is addressed in Standard 7.07 of the APA’s Ethics Code and Principle II.28 of the Canadian Code of Ethics. Standard 7.07 states: “Psychologists do not engage in sexual relationships with students or supervisees who are in their department, agency, or training center or over whom psychologists have or are likely to have evaluative authority.”

The correct answer is: is prohibited when the psychologist currently has or is likely to have evaluative authority over the student.

How well did you know this?
1
Not at all
2
3
4
5
Perfectly
63
Q

Obsessive-Compulsive Disorder and Obsessive-Compulsive Personality Disorder share which of the following symptoms?
Select one:

A.
prominent obsessions

B.
repetitive rituals

C.
risk aversion

D.
emotional rigidity

A

Answer B is correct: Both disorders involve rituals, but the goal of rituals is different for OCD and OCPD. In OCD, rituals are performed to reduce anxiety; in OCPD, rituals are related to perfectionism.

Answer A: Prominent obsessions are characteristic of OCD but not OCPD.

Answer C: Risk aversion is not a characteristic shared by the two disorders.

Answer D: Emotional rigidity is not a characteristic shared by OCD and OCPD.

The correct answer is: repetitive rituals

How well did you know this?
1
Not at all
2
3
4
5
Perfectly
64
Q

In factor analysis, “rotating” the factors has which of the following effects?
Select one:

A.
It changes the factor loadings for the variables and the eigenvalue for each factor.

B.
It changes the factor loadings for the variables but does not change the eigenvalue for each factor.

C.
It changes the eigenvalue for each factor but does not change the factor loadings for the variables.

D.
It does not change the eigenvalue for each factor or the factor loadings for the variables.

A

The purpose of rotation in factor analysis is to facilitate interpretation of the factors.
a. CORRECT Rotation alters the factor loadings for each variable and the eigenvalue for each factor (although the total of the eigenvalues remains the same). Knowing that an eigenvalue indicates the amount of variability accounted for by each factor may have helped you identify the correct answer to this question – i.e., when the factor loadings change, the eigenvalues will also change.

b. Incorrect See explanation above.
c. Incorrect See explanation above.
d. Incorrect See explanation above.

The correct answer is: It changes the factor loadings for the variables and the eigenvalue for each factor.

How well did you know this?
1
Not at all
2
3
4
5
Perfectly
65
Q

A father is trying to work in the garden, but his son wants his attention and keeps whining and nagging. The father finally gives in and plays with the boy who stops whining. The father’s behavior (playing with his son) is the result of:
Select one:

A.
positive punishment.

B.
negative punishment.

C.
positive reinforcement.

D.
negative reinforcement.

A

The father’s play behavior has increased in this situation, which means that it is being reinforced.
a. Incorrect See explanation for response d.

b. Incorrect See explanation for response d.
c. Incorrect See explanation for response d.
d. CORRECT When the father plays with the child, the child stops nagging and whining. In other words, the father’s play behavior is being negatively reinforced.

The correct answer is: negative reinforcement.

How well did you know this?
1
Not at all
2
3
4
5
Perfectly
66
Q

Which of the following best describes a psychologist’s ethical obligation with regard to obtaining an informed consent from a research participant?
Select one:

A.
An informed consent is always required.

B.
An informed consent may be unnecessary when the study is unlikely to cause the participant harm or distress.

C.
An informed consent is unnecessary only when the study involves an anonymous survey or questionnaire or naturalistic observation.

D.
An informed consent is unnecessary only when the study involves deception and the deception meets ethical requirements.

A

Informed consents are addressed in Standards 8.02 and 8.05 of the APA’s Ethics Code and Principles I.19 and I.20 of the Canadian Code of Ethics.
a. Incorrect Informed consents are not always required.

b. CORRECT The potential for harm is a primary consideration when determining whether an informed consent is needed.
c. Incorrect An informed consent may not be necessary in the situations described in this answer. However, these are not the “only” situations that do not require an informed consent, so this is not the best answer.
d. Incorrect An informed consent cannot be obtained when a study involves deception, but there are other circumstances when an informed consent is not necessary, so this is not the best response.

The correct answer is: An informed consent may be unnecessary when the study is unlikely to cause the participant harm or distress.

How well did you know this?
1
Not at all
2
3
4
5
Perfectly
67
Q

Kaila K., age 12, is brought to therapy by her court-appointed legal guardian who says she is concerned because Kaila has recently started getting into fights with her peers and is acting disrespectfully toward her teacher. The therapist should:
Select one:

A.
see the child in therapy as requested by the guardian.

B.
see the child in therapy only after obtaining permission from one of the child’s biological parents.

C.
see the child in therapy only after obtaining permission from one of the child’s biological relatives.

D.
see the child in therapy only after obtaining permission from the court.

A

A child’s legal guardian has the authority to make decisions relevant to the child’s well-being.
a. CORRECT The legal guardian may authorize treatment for the child, and approval from the court or a biological relative of the child is unnecessary.

b. Incorrect See explanation above.
c. Incorrect See explanation above.
d. Incorrect See explanation above.

The correct answer is: see the child in therapy as requested by the guardian.

How well did you know this?
1
Not at all
2
3
4
5
Perfectly
68
Q

Fathers who have a child who has received a diagnosis of ADHD:
Select one:

A.
tend to rate the child’s behaviors as less problematic than mothers do.

B.
tend to rate the behaviors of their son (but not daughter) with ADHD as less problematic than mothers do.

C.
tend to rate the behaviors of their daughter (but not son) with ADHD as less problematic than mothers do.

D.
tend to rate the child’s behaviors as more problematic than mothers do.

A

This is a difficult question because very few of the existing studies consider the reactions of fathers of children with ADHD.
a. CORRECT The limited research on this issue suggests that fathers of children with ADHD rate the behaviors of their children as less problematic. For example, E. J. Mash and C. Johnston found that fathers tend to rate the behaviors of children with ADHD as less troublesome than mothers (Parental perceptions of child behavior problems, parenting self-esteem, and mothers’ reported stress in younger and older hyperactive and normal children, Journal of Consulting and Clinical Psychology, 51, 86-99, 1983). And, more recently, Singh interviewed 39 mothers and 22 fathers of boys who had received the diagnosis of ADHD and found that only four of the fathers felt their sons’ behaviors required medical attention [Boys will be boys: Fathers’ perspectives on ADHD symptoms, diagnosis, and drug treatment, Harvard Review of Psychiatry, 11(6), 308-316, 2003].

The correct answer is: tend to rate the child’s behaviors as less problematic than mothers do.

How well did you know this?
1
Not at all
2
3
4
5
Perfectly
69
Q

Test-score banding is used to:
Select one:

A.
alleviate criterion contamination.

B.
equate an examinees scores on different tests.

C.
facilitate criterion-referenced interpretation.

D.
reduce adverse impact.

A

Test-score banding is a method of score interpretation that involves considering people who obtain scores within a specific range (band) as having equivalent scores.
a. Incorrect See explanation for response d.

b. Incorrect See explanation for response d.
c. Incorrect See explanation for response d.
d. CORRECT Banding has been identified as a method for reducing biases in testing that contribute to group differences in test scores. Its advocates argue that it is an effective way for reducing adverse impact.

The correct answer is: reduce adverse impact.

How well did you know this?
1
Not at all
2
3
4
5
Perfectly
70
Q

Research suggests that, with regard to metamemory, older adults are:
Select one:

A.
more likely than younger adults to underestimate their actual memory deficits.

B.
more likely than younger adults to overestimate their actual memory deficits.

C.
just as able as younger adults to accurately estimate their actual memory deficits.

D.
better than younger adults at estimating their actual memory deficits.

A

The research on this topic is limited, but it appears that while older adults do, in fact, experience memory problems, they may overestimate the extent of those problems.
a. Incorrect See explanation for response b.

b. CORRECT Metamemory refers to the ability to monitor one”s own memory processes. There is evidence that, compared to younger adults, older adults tend to view their memory loss as worse than it actually is.
c. Incorrect See explanation for response b.
d. Incorrect See explanation for response b.

The correct answer is: more likely than younger adults to overestimate their actual memory deficits.

How well did you know this?
1
Not at all
2
3
4
5
Perfectly
71
Q

In comparison to a non-hypnotic state, hypnosis is most likely to produce:
Select one:

A.
more memories in general with a smaller proportion of false memories.

B.
more memories in general with a larger proportion of false memories.

C.
fewer memories in general with a smaller proportion of false memories.

D.
fewer memories in general with a larger proportion of false memories.

A

Research conducted in the past decade has not been very supportive of the usefulness of hypnosis for retrieving repressed memories.
a. Incorrect See explanation for response b.

b. CORRECT The state of hypnosis fosters the production of information that would normally not be reported with much of that information being confabulated.
c. Incorrect See explanation for response b.
d. Incorrect See explanation for response b.

The correct answer is: more memories in general with a larger proportion of false memories.

How well did you know this?
1
Not at all
2
3
4
5
Perfectly
72
Q

Which of the following best describes research findings on the impact of serotonin levels on the eating behaviors of individuals with an Eating Disorder?
Select one:

A.
Low levels of serotonin are believed to precipitate binge eating in Bulimia and starvation in Anorexia.

B.
High levels of serotonin are believed to precipitate binge eating in Bulimia and starvation in Anorexia.

C.
High levels of serotonin are believed to precipitate starvation in Anorexia while low levels are believed to precipitate binge eating in Bulimia.

D.
Low levels of serotonin are believed to precipitate starvation in Anorexia while high levels are believed to precipitate binge eating in Bulimia.

A

Answer C is correct: High levels of serotonin have been linked to both appetite suppression and anxiety. Reduced caloric intake by those with Anorexia apparently fosters a sense of calm and personal control by lowering serotonin levels. Low levels of serotonin have been linked to depression and apparently contribute to binge eating in Bulimia - i.e., bingeing on sweets and carbohydrates increases serotonin levels and thereby elevates mood.

Answers A, B, and D: See explanation for answer C.

The correct answer is: High levels of serotonin are believed to precipitate starvation in Anorexia while low levels are believed to precipitate binge eating in Bulimia.

How well did you know this?
1
Not at all
2
3
4
5
Perfectly
73
Q

Many antipsychotic drugs are believed to exert their therapeutic effects primarily by blocking dopamine receptors (especially D2 receptors) in the brain. An exception is:
Select one:

A.
haloperidol.

B.
fluphenazine.

C.
chlorpromazine.

D.
clozapine.

A

Of the antipsychotic drugs listed in the responses, only one is considered to be atypical because of its effects on the nervous system.
a. Incorrect See explanation for response d.

b. Incorrect See explanation for response d.
c. Incorrect See explanation for response d.
d. CORRECT Clozapine seems to have limited effects on D2 receptors and, instead, affects D4 and other dopamine receptors as well as serotonin and glutamate receptors.

The correct answer is: clozapine.

How well did you know this?
1
Not at all
2
3
4
5
Perfectly
74
Q

Encoding strategies, strategy construction, generalization, and automatization are of most interest to:
Select one:

A.
Piagetian theorists.

B.
Vygotskian theorists.

C.
information processing theorists.

D.
neo-Piagetian theorists.

A

Recognition that the terms listed in the question are cognitive processes may have helped you choose the correct answer to this question even if you are not familiar with all of the theories listed.
a. Incorrect See explanation for response c.

b. Incorrect See explanation for response c.
c. CORRECT Information-processing theorists focus on the processes of the mind including those listed in this question (which are categorized as information processing “change mechanisms”).
d. Incorrect See explanation for response c.

The correct answer is: information processing theorists.

How well did you know this?
1
Not at all
2
3
4
5
Perfectly
75
Q

A better prognosis for Schizophrenia is associated with all of the following except:
Select one:

A.
female gender.

B.
an early onset of symptoms.

C.
a family history of a mood disorder.

D.
the presence of a precipitating event.

A

Answer B is correct: A late (not early) onset of symptoms is associated with a better prognosis.

Answers A, C, and D: Female gender, a family history of a mood disorder, and the presence of a precipitating event have been linked to a better prognosis.

The correct answer is: an early onset of symptoms.

How well did you know this?
1
Not at all
2
3
4
5
Perfectly
76
Q

The item difficulty index (p) represents which scale of measurement?
Select one:

A.
ordinal

B.
interval

C.
nominal

D.
ratio

A

The item difficulty index (p) is calculated by dividing the number of individuals who answered the item correctly by the total number of individuals.
a. CORRECT To understand why the item difficulty index represents an ordinal scale, assume that items 1, 2, and 3 of a test are passed by 10, 20, and 30 percent of examinees, respectively, which will result in p values for these items of .10, .20, and .30. Although these values indicate that item 1 is more difficult than item 2 which, in turn, is more difficult than item 3, it is not possible to say that item 2 is twice as difficult as item 1 or that the difference in difficulty between items 1 and 2 is equal to the difference between items 2 and 3. Moreover, an item difficulty index of 0 would not mean that the item completely lacks difficulty (which doesn”t really make any sense). In other words, p values represent an ordinal scale because they do not have the property of equal intervals or an absolute zero point.

b. Incorrect See explanation for response a.
c. Incorrect See explanation for response a.
d. Incorrect See explanation for response a.

The correct answer is: ordinal

How well did you know this?
1
Not at all
2
3
4
5
Perfectly
77
Q

When developing an achievement test, it is important to keep in mind that an average item difficulty index of _____ will help ensure that scores on the test are normally distributed and that discrimination between examinees with high and low ability will be maximized.
Select one:

A.
10

B.
1

C.
0.5

D.
0

A

The item difficulty index (p) ranges from 0 to 1.0, with 0 indicating a very easy item and 1.0 indicating a very difficult item.

c. CORRECT For most tests, an average item difficulty level of .50 (moderate difficulty) is preferred for several reasons including those listed in this question.

The correct answer is: 0.5

How well did you know this?
1
Not at all
2
3
4
5
Perfectly
78
Q

When treating a patient with Delirium, a priority is to identify and then treat or remove its cause. In addition, it is important to:
Select one:

A.
make sure that the patient is provided with adequate stimulation.

B.
keep the patient in a quiet room where he/she can be monitored by a family or staff member.

C.
administer a neuroleptic to reduce the patient’s disorientation and agitation.

D.
make sure family members and friends visit regularly.

A

Answer B is correct: Delirium is characterized by disorientation and confusion, so an important goal is provide an environment that decreases disorientation. For example, it is important to reduce distractions and to provide a quiet, well-lit room and constant monitoring.

Answer A: Although “adequate stimulation” could be taken to mean stimulation that is appropriate for a disoriented state, this response is too vague and not as good as answer B.

Answer C: Neuroleptics may or may not be appropriate, depending on the cause of the Delirium.

Answer D: With a delirious patient, you want to provide a consistent environment, so regular visits by different people would probably be contraindicated.

The correct answer is: keep the patient in a quiet room where he/she can be monitored by a family or staff member.

How well did you know this?
1
Not at all
2
3
4
5
Perfectly
79
Q

When the target behavior occurs very infrequently or leaves a permanent record, the best observational technique is:
Select one:

A.
interval recording.

B.
time sampling.

C.
event recording.

D.
situational sampling.

A

The strategies listed in the answers are different types of behavioral observation techniques.
a. Incorrect Interval recording is useful for behaviors that have no clear beginning or end and involves dividing the time period into intervals and recording whether or not the behavior occurred during each interval.

b. Incorrect Time sampling involves checking to see if a behavior occurs during predetermined points in time and is useful for monitoring high-rate continuous behaviors.
c. CORRECT Event recording entails observing a behavior each time it occurs and is useful for infrequent behaviors and behaviors that leave a permanent record.
d. Incorrect Situational sampling involves observing a behavior in a number of different settings.

The correct answer is: event recording.

How well did you know this?
1
Not at all
2
3
4
5
Perfectly
80
Q

To assess the effectiveness of differential reinforcement for alleviating a child’s bad habits (i.e., nail biting, thumb-sucking, hair-pulling, and pencil-chewing), the best research design would be which of the following?
Select one:

A.
time-series

B.
Solomon four-group

C.
multiple baseline

D.
reversal

A

In this situation, you want to test the effectiveness of an intervention for four different behaviors, and your study will include only one participant.
a. Incorrect A time-series design is a group design that involves measuring the dependent variable multiple times before and after the intervention is applied.

b. Incorrect The Solomon four-group design is used to evaluate the effects of pretesting on the dependent variable.
c. CORRECT The multiple baseline design is a single-subject design that can be used to assess a treatment for different behaviors, settings, or participants. Additional information about the multiple-baseline design is provided in the Statistics and Research Design chapter of the written study materials.
d. Incorrect The reversal design is also a single-subject design but is not as useful as the multiple baseline design for assessing the effectiveness of a treatment for different behaviors.

The correct answer is: multiple baseline

How well did you know this?
1
Not at all
2
3
4
5
Perfectly
81
Q

Which of the following is NOT a DSM-5 diagnostic criterion for Bulimia Nervosa?
Select one:

A.
Binges are accompanied by a sense of a lack of control over eating.

B.
Binges are accompanied by inappropriate compensatory behaviors to prevent weight gain.

C.
People with this disorder have an intense fear of gaining weight and becoming fat.

D.
The self-esteem of people with this disorder is unduly affected by their body shape and weight.

A

Answer C is correct: Intense fear of gaining weight and becoming fat is a diagnostic criterion for Anorexia Nervosa but not for Bulimia Nervosa.

Answers A, B, and D: See explanation for answer C.

The correct answer is: People with this disorder have an intense fear of gaining weight and becoming fat.

How well did you know this?
1
Not at all
2
3
4
5
Perfectly
82
Q

With regard to terminating the life of an animal used in research, the ethics codes of the American and Canadian Psychological Associations:
Select one:

A.
prohibit this practice except in the “most unusual circumstances.”

B.
permit this practice only when the animal is experiencing pain as the result of the experimental procedures.

C.
permit this practice only with the approval of the institutional review board.

D.
permit this practice when it is done in a humane manner.

A

This issue is addressed by Standard 8.09(g) of the APA’s Ethics Code and Principle II.47 of the Canadian Code of Ethics.
a. Incorrect See explanation for response d.

b. Incorrect See explanation for response d.
c. Incorrect See explanation for response d.
d. CORRECT Standard 8.09(g) of the APA’s Ethics Code states: “When it is appropriate that an animal’s life be terminated, psychologists proceed rapidly, with an effort to minimize pain and in accordance with accepted procedures.”

The correct answer is: permit this practice when it is done in a humane manner.

How well did you know this?
1
Not at all
2
3
4
5
Perfectly
83
Q

According to Erikson, ________ is the basic strength associated with the ego integrity vs. despair stage of psychosocial development.
Select one:

A.
courage

B.
fidelity

C.
wisdom

D.
care

A

Erikson distinguished between eight stages of psychosocial development and defined positive outcomes for each stage in terms of one or more basic strengths.
a. Incorrect Courage is a basic strength of the autonomy vs. shame stage.

b. Incorrect Fidelity is a basic strength of the identity vs. role confusion stage.
c. CORRECT Wisdom is the basic strength associated with the final ego integrity vs. despair stage of development.
d. Incorrec Care is a basic strength of the generativity vs. stagnation stage.

The correct answer is: wisdom

How well did you know this?
1
Not at all
2
3
4
5
Perfectly
84
Q

Dr. Greg Gallant meets 28-year-old Sally S. at a holiday party and is sexually attracted to her. The feelings seem mutual. However, during their conversation, he learns that Sally’s sister is one of his therapy clients. In this situation:
Select one:

A.
it would be acceptable for Dr. Gallant to date Sally.

B.
it would be acceptable for Dr. Gallant to date Sally only if her sister is a former therapy client.

C.
it would be acceptable for Dr. Gallant to date Sally only if he discusses the implications with the client before doing so.

D.
it would be acceptable for Dr. Gallant to date Sally only if he believes that doing so will not be harmful to his client.

A

This issue is addressed in Standard 10.06 of the APA’s Ethics Code and Principle III.31 of the Canadian Code of Ethics for Psychologists.
a. Incorrect See explanation for response b.

b. CORRECT The Ethics Code states that “psychologists do not engage in sexual intimacies with individuals they know to be close relatives, guardians, or significant others of current therapy clients/patients,” which implies that it may be acceptable to become involved with a relative of a former therapy client. The Canadian Code of Ethics does not explicitly distinguish between relatives of current versus former clients, but this is still the best answer of those given since the other answers state or imply that the sister is a current client.
c. Incorrect See explanation for response b.
d. Incorrect See explanation for response b.

The correct answer is: it would be acceptable for Dr. Gallant to date Sally only if her sister is a former therapy client.

How well did you know this?
1
Not at all
2
3
4
5
Perfectly
85
Q

Dr. Best is attempting to set the optimal cutoff score for a new screening test designed to identify people at risk for drug abuse. Using the data he collected when evaluating the test’s criterion-related validity, he finds that lowering the cutoff score on the screening test:
Select one:

A.
increases the number of true positives and true negatives.

B.
decreases the number of true positives and true negatives.

C.
increases the number of true and false positives.

D.
decreases the number of true and false positives.

A

Lowering the predictor (screening device) cutoff score increases the number of “positives” and decreases the number of “negatives.”
a. Incorrect See explanation for response c.

b. Incorrect See explanation for response c.
c. CORRECT “Positives” are individuals who are identified by a predictor as having the attribute being assessed - in this case, people who are at risk for drug abuse. Lowering the predictor cutoff increases the number of true and false positives.
d. Incorrect See explanation for response c.

The correct answer is: increases the number of true and false positives.

How well did you know this?
1
Not at all
2
3
4
5
Perfectly
86
Q

The children of parents who are ______________ tend to be self-confident, socially responsible, and achievement-oriented.
Select one:

A.
low in demandingness and low in warmth

B.
low in demandingness and high in warmth

C.
high in demandingness and low in warmth

D.
high in demandingness and high in warmth

A

The research has linked authoritative parenting to the most positive outcomes for children and adolescents.
a. Incorrect This combination of demandingness and warmth describes rejecting-neglecting parents.

b. Incorrect This combination describes permissive parents.
c. Incorrect This combination is characteristic of authoritarian parents.
d. CORRECT Authoritative parents are high in demandingness and high in warmth (responsivity). The children of these parents tend to be better adjusted than children of rejecting-neglecting, permissive, or authoritarian parents.

The correct answer is: high in demandingness and high in warmth

How well did you know this?
1
Not at all
2
3
4
5
Perfectly
87
Q

As part of a peer review, an insurance company requests that you provide it with information about a client whose fee is being paid in part by the insurance company. In this situation, you should:
Select one:

A.
release information to the company only after determining that company personnel are aware of the importance of maintaining the clients confidentiality.

B.
release information to the company only after obtaining a waiver from the client.

C.
release information to the company as requested since regulations regarding client confidentiality do not apply to this situation.

D.
refuse to release information to the company.

A

A psychologist is required to provide the requested information for peer reviews.
a. CORRECT Of the answers given, this is the best one. Although psychologists are required to provide an insurance company with the information it has requested for a peer review, the best course of action would be to remind the company of the need to maintain the information in a way that will minimize violations of the client’s confidentiality.

b. Incorrect In this situation, you do not need to obtain a release from the client.
c. Incorrect A psychologist’s legal and ethical obligations related to client confidentiality are not entirely waived for peer reviews.
d. Incorrect As noted above, psychologists are required to provide requested information for peer reviews.

The correct answer is: release information to the company only after determining that company personnel are aware of the importance of maintaining the clients confidentiality.

How well did you know this?
1
Not at all
2
3
4
5
Perfectly
88
Q

Sperling (1960) had research participants stare at a blank screen onto which he flashed a display of letters for a fraction of a second and then asked them to recall as many of the letters as they could. Sperling was studying which of the following?
Select one:

A.
eidetic memory

B.
semantic memory

C.
iconic memory

D.
echoic memory

A

For the exam, you want to be familiar with the four terms listed in the answers to this question.
a. Incorrect Eidetic memory is also known as photographic memory and refers to visual images that are particulary vivid and detailed. Although Sperling was studying memory for visual images, this is not the best answer of those given.

b. Incorrect Semantic memory is the aspect of long-term memory that includes general knowledge.
c. CORRECT Iconic memory is the aspect of sensory memory that stores visual stimuli. Sperling was studying iconic memory.
d. Incorrect Echoic memory is the aspect of sensory memory that stores auditory stimuli.

The correct answer is: iconic memory

How well did you know this?
1
Not at all
2
3
4
5
Perfectly
89
Q

Which of the following best describes the difference between trauma-induced retrograde and anterograde amnesia?
Select one:

A.
Retrograde amnesia involves memory loss for events occurring before the trauma, while anterograde amnesia involves memory loss for events occurring after the trauma.

B.
Retrograde amnesia involves memory loss for events occurring after the trauma, while anterograde amnesia involves memory loss for events occurring at the time of the trauma.

C.
Retrograde amnesia involves memory loss for events that occurred in the distant past, while anterograde amnesia involves memory loss for events that occurred just before the trauma.

D.
Retrograde amnesia involves memory loss for events occurring after the trauma, while anterograde amnesia involves memory loss for events occurring before the trauma.

A

Amnesia (loss of memory) is categorized as anterograde or retrograde.
a. CORRECT This is a correct description of retrograde and anterograde amnesia: Retrograde amnesia is a loss of memory for events that occur before the trauma, while anterograde amnesia is a loss of memory for events that occur subsequent to the trauma.

b. Incorrect See explanation for response a.
c. Incorrect See explanation for response a.
d. Incorrect See explanation for response a.

The correct answer is: Retrograde amnesia involves memory loss for events occurring before the trauma, while anterograde amnesia involves memory loss for events occurring after the trauma.

How well did you know this?
1
Not at all
2
3
4
5
Perfectly
90
Q

According to Albert Ellis, our emotional and behavioral reactions to an event are due to our beliefs about the event rather than to the event itself. In other words, our beliefs act as a:
Select one:

A.
moderator variable.

B.
mediator variable.

C.
latent variable.

D.
suppressor variable.

A

Ellis’s theory proposes that beliefs mediate (are responsible for) the impact of an event on our emotional and behavioral reactions to that event.
a. Incorrect A moderator variable is a variable that affects the strength of the relationship between two other variables. For example, if the size of the correlation between a predictor and criterion differs for older and younger adults, age is a moderator variable. Note that some authors use the terms “moderator variable” and “extraneous variable” interchangeably.

b. CORRECT A mediator variable accounts for (is responsible for) the relationship between two variables.
c. Incorrect A latent variable is a theoretical variable that is believed to underlie a measured or observed variable.
d. Incorrect A suppressor variable reduces or conceals the relationship between two variables. Consequently, statistically removing the effects of a suppressor variable increases the correlation between the two variables.

The correct answer is: mediator variable.

How well did you know this?
1
Not at all
2
3
4
5
Perfectly
91
Q

A consultant is hired to help a teacher who has been working at an inner-city school for two years. The teacher feels she does not have the skills she needs to deal with ongoing race-related conflicts that she often encounters in her classroom. This is an example of:
Select one:

A.
client-centered case consultation.

B.
consultee-centered case consultation.

C.
program-centered administrative consultation.

D.
consultee-centered administrative consultation.

A

In this situation, the consultant’s goal is to help the teacher acquire the skills she needs to deal with ongoing problems in her classroom.
a. Incorrect In client-centered case consultation, there would be a particular student who would be the focus of the consultation.

b. CORRECT In consultee-centered case consultation, the focus in on the consultee’s skills, abilities, objectivity, etc.
c. Incorrect In program-centered administrative consultation, an existing program is the focus.
d. Incorrect In this type of consultation, the consultant helps administrators acquire the skills they need for future program development and implementation.

The correct answer is: consultee-centered case consultation.

92
Q

To calculate the F-ratio for a one-way ANOVA:
Select one:

A.
MSB is multiplied by MSW.

B.
MSB is divided by MSW.

C.
MSW is divided by MSB.

D.
MSW is subtracted from MSB.

A

As described in the Statistics and Research Design chapter of the written study materials, the F-ratio is calculated by dividing a measure of treatment plus error by a measure of error only in order to obtain an estimate of treatment effects.
a. Incorrect See explanation for response b.

b. CORRECT MSB (mean square between) is a measure of treatment plus error, while MSW (mean square within) is a measure of error only, and the F-ratio is calculated by dividing MSB by MSW.
c. Incorrect See explanation for response b.
d. Incorrect See explanation for response b.

The correct answer is: MSB is divided by MSW.

93
Q

Children with Tourette’s Disorder often have symptoms of ADHD. In addition, they most often exhibit which of the following?
Select one:

A.
obsessions and compulsions

B.
separation anxiety

C.
conduct problems

D.
below-average IQ

A

Answer A is correct: The most common comorbid symptoms for Tourette’s Disorder are obsessions and compulsions and symptoms of ADHD.

Answers B, C, and D: See explanation for answer A.

The correct answer is: obsessions and compulsions

94
Q

Bowlby (1980) proposed that a child’s early experiences lead to the development of _________, which then affects his or her future relationships.
Select one:

A.
object permanence

B.
gender identity

C.
an internal working model

D.
a transactional model

A

According to Bowlby, a child’s early experiences with primary caregivers leads to the development of an internal working model, which is a mental representation of oneself and others and relationships between oneself and others.

a. Incorrect See explanation for answer c.
b. Incorrect See explanation for answer c.
c. CORRECT A child’s internal working model influences his or her future relationships.
d. Incorrect See explanation for answer c.

The correct answer is: an internal working model

95
Q

As defined by Herek (2004), __________ promotes hostility and violence toward homosexuals and consists of beliefs about gender, identity, and sexuality that define sexual minorities as deviant or threatening.
Select one:

A.
homophobia

B.
sexual stigma

C.
sexual prejudice

D.
heterosexism

A

For the exam, you want to be familiar with Herek’s definitions for the four terms listed in the answers to this question. These definitions are provided in the Clinical Psychology chapter of the written study materials.
a. Incorrect See explanation for response d.

b. Incorrect See explanation for response d.
c. Incorrect See explanation for response d.
d. CORRECT Herek considers homophobia to be an imprecise term and recommends that it be replaced with the terms listed in answers b, c, and d. The description in this question is consistent with Herek’s definition of heterosexism.

The correct answer is: heterosexism

96
Q

When a woman experiences chronic stress and anxiety during her pregnancy, there is a greater risk for all of the following except:
Select one:

A.
premature birth of the infant.

B.
a greater number of delivery complications.

C.
an infant with lower-than-normal levels of activity.

D.
an infant with more feeding and sleep problems than normal.

A

Maternal stress is one of a number of prenatal environmental factors that influences child development. Others include maternal health, diet, and drug use.
a. Incorrect See explanation for response c.

b. Incorrect See explanation for response c.
c. CORRECT High levels of maternal stress during pregnancy is associated with prematurity, abortion, delivery complications, infants who are hyperactive and irritable, and infants who have feeding and sleep problems.
d. Incorrect See explanation for response c.

The correct answer is: an infant with lower-than-normal levels of activity.

97
Q

In most people, the left hemisphere is dominant for which of the following functions?
Select one:

A.
perceiving whole shapes from parts

B.
recognizing emotions in facial expressions

C.
perceiving direction and distance

D.
controlling complex and precise movements

A

Knowing that the right (nondominant) hemisphere is responsible for processing emotional and spatial information would have helped you eliminate responses a, b, and c as correct answers to this question.
a. Incorrect The right hemisphere is responsible for understanding part-whole relationships - for example, recognizing familiar objects from incomplete pictorial data and perceiving whole shapes from parts.

b. Incorrect The right hemisphere is also responsible for decoding emotion in facial expressions and language.
c. Incorrect The right hemisphere predominates in the perception of depth, distance, direction, and orientation.
d. CORRECT In the majority of people, the left hemisphere is dominant for fine motor functioning, which is evident in the fact that most people are right-handed.

The correct answer is: controlling complex and precise movements

98
Q

A researcher is studying the ability of college students to remember lists of related word pairs (e.g., dog-cat, chair-table). Prior to studying the to-be-remembered list of word pairs, the experimental group is presented with a list of opposite word pairs such as tall-short, while the control group completes multiplication problems. What results should the researcher expect when the two groups are subsequently asked to recall the word pairs?
Select one:

A.
Due to retroactive interference, the control group will recall more word pairs.

B.
Due to retroactive interference, the experimental group will recall more word pairs.

C.
Due to proactive interference, the experimental group will recall fewer word pairs.

D.
Due to proactive interference, the control group will recall fewer word pairs.

A

This question describes a standard proactive interference study. During the first phase, the experimental group is presented with a list of word pairs (e.g., tall-short) and the control group engages in an unrelated arithmetic task. During the second phase, the experimental and control groups both study the to-be-remembered word pairs (e.g., dog-cat). During the third phase, participants in both groups are asked to recall the word pairs.
a. Incorrect See explanation for response c.

b. Incorrect See explanation for response c.
c. CORRECT For the experimental group, the two tasks are similar, and the first list of word pairs interferes with the ability to learn or recall the second list. This is referred to as proactive interference. For the control group, the two tasks are dissimilar, and there is less interference. Consequently, the control group will remember more words than the experimental group.
d. Incorrect See explanation for response c.

The correct answer is: Due to proactive interference, the experimental group will recall fewer word pairs.

99
Q

In the Strange Situation, a child with ________ attachment stays close to his/her mother before they are separated but, when she returns following a separation, is angry and resistant and may push his/her mother away and is not easily comforted.
Select one:

A.
disorganized/disoriented

B.
insecure/avoidant

C.
insecure/ambivalent

D.
insecure/distrustful

A

Four attachment patterns were identified by Ainsworth and her colleagues: secure, insecure/avoidant, insecure/ambivalent, and disorganized/disoriented. See the Lifespan Development chapter for information about these patterns.
a. Incorrect See explanation for response c.

b. Incorrect See explanation for response c.
c. CORRECT As its name implies, the insecure/ambivalent pattern is characterized by ambivalence – e.g., a child with this pattern will both seek closeness to his/her parent and push the parent away. Note that this pattern is also referred to as insecure/resistant attachment.
d. Incorrect See explanation for response c.

The correct answer is: insecure/ambivalent

100
Q

Broadbent (1958) used the dichotic listening task to investigate the predictions of his filter theory of attention. This task involved having the listener hear different messages presented simultaneously to each ear and then verbally report what he/she had heard. For example, in a typical study, three digits (e.g., 5, 2, 7) were presented to one ear, while three other digits (e.g., 4, 9, 8) were simultaneously presented to the other ear. The results of these studies confirmed Broadbent’s theory because, in most cases, listeners reported that they had heard which of the following?
Select one:

A.
5-4-2-9-7-8

B.
5-2-7-4-9-8

C.
5-2-7 only OR 4-9-8 only

D.
7-8 only

A

Broadbent’s (1958) filter theory of attention was the first comprehensive theory of attention and the first of the “bottleneck” theories. It proposes that a filter (sensory buffer) selects which incoming message will be further processed or attended to on the basis of the physical properties of the message.
a. Incorrect See explanation for response b.

b. CORRECT Most participants in Broadbent’s studies reported the information by ear rather than by the order in which they heard the digits, and he concluded that this proved that the physical properties of the message determined if it was attended to or ignored. In other words, in the dichotic listening task studies, the filter allowed the message from one ear to “pass through” before it allowed the message from the other ear to do so, which explained why participants would report the digits heard in one ear first, followed by the digits heard in the other ear.
c. Incorrect See explanation for response b.
d. Incorrect See explanation for response b.

The correct answer is: 5-2-7-4-9-8

101
Q

With regard to fees in forensic settings, the APA’s Specialty Guidelines for Forensic Psychology:
Select one:

A.
states that psychologists should avoid providing services to participants in a legal proceeding on the basis of “contingent fees.”

B.
states that psychologists should avoid providing services to participants in a legal proceeding on the basis of “contingent fees” except when those services are court-ordered.

C.
recommends that psychologists not provide services to participants involved in a legal proceeding on the basis of “contingent fees” except when such fees are compatible with current standards of practice.

D.
does not explicitly address the issue of providing services to participants in a legal proceeding on the basis of “contingent fees.”

A

Contingent fees are addressed in Paragraph 5.02 of Specialty Guidelines for Forensic Psychology (APA, 2012).
a. CORRECT Paragraph 5.02 states: “Because of the threat to impartiality presented by the acceptance of contingent fees and associated legal prohibitions, forensic practitioners strive to avoid providing professional services on the basis of contingent fees.”

b. Incorrect See explanation for response a.
c. Incorrect See explanation for response a.
d. Incorrect See explanation for response a.

The correct answer is: states that psychologists should avoid providing services to participants in a legal proceeding on the basis of “contingent fees.”

102
Q

You are conducting a multiple regression analysis and find that one of your predictors has a negative regression coefficient. This means that:
Select one:

A.
you should eliminate that predictor from the regression equation.

B.
the predictor has an inverse relationship with the criterion.

C.
the predictor is uncorrelated with the other predictors included in the analysis.

D.
you have made a mistake in your data entry or calculations.

A

The size and sign of a predictor’s regression coefficient are related to the nature of the correlation between the predictor and criterion.
a. Incorrect A negative sign does not mean there is anything wrong with the predictor.

b. CORRECT A negative sign means that there is an inverse (negative) correlation between the predictor and the criterion.
c. Incorrect This is not implied by a negative regression coefficient.
d. Incorrect A negative regression coefficient is a possibility and, therefore, is not necessarily indicative of a data entry or calculation error.

The correct answer is: the predictor has an inverse relationship with the criterion.

103
Q

Which of the following studies confirmed the conclusion drawn by Sherif on the basis of the results of his “Robber’s Cave” study?
Select one:

A.
Milgram’s study on obedience to authority

B.
Bandura’s work on imitation and aggression

C.
Kelly’s research on the discounting principle

D.
Aronson’s research on the jigsaw classroom

A

Sherif found that hostility between competing groups was reduced by having members of the groups work together to achieve a superordinate goal.
a. Incorrect See explanation for response d.

b. Incorrect See explanation for response d.
c. Incorrect See explanation for response d.
d. CORRECT In a jigsaw classroom, students work on a project as a team and are therefore dependent on one another to complete the group assignment. The research has shown that the jigsaw classroom is useful for reducing prejudice and hostility.

The correct answer is: Aronson’s research on the jigsaw classroom

104
Q

Which of the following is NOT a condition for a claim of malpractice?
Select one:

A.
legal duty of care

B.
breach of duty

C.
lack of consent

D.
causation

A

For the exam, you want to be familiar with the four conditions for malpractice.
a. Incorrect See explanation for response c.

b. Incorrect See explanation for response c.
c. CORRECT The four conditions for malpractice are: (1) a legal duty to the patient; (2) a breach or dereliction of that duty; (3) damage to the patient; and (4) a causal relationship between the breach of duty and the damage.
d. Incorrect See explanation for response c.

The correct answer is: lack of consent

105
Q

Dr. Ronald R. has been dating a young woman he met through an online dating service for two months and their relationship is starting to get serious. However, on the last date, he learned that his new girlfriend is the sister of one of his therapy clients. The client is an adult and does not live with his sister. As an ethical psychologist, Dr. R. should:
Select one:

A.
stop seeing the sister until his therapeutic relationship with the client has terminated.

B.
terminate therapy with the client after providing him with appropriate referrals.

C.
discuss the matter with the client as soon as possible to help determine the best course of action.

D.
do nothing unless it becomes evident that the situation is reducing his effectiveness as a therapist.

A

This situation can be interpreted as constituting a multiple relationship and, therefore, requires that some action be taken.
a. Incorrect See explanation for response c.

b. Incorrect See explanation for response c.
c. CORRECT The situation described in this question is not directly addressed by the APA’s Ethic Code or the Canadian Code of Ethics, but this response is most consistent with their requirements regarding multiple relationships. For example, Standard 3.05(b) of the Ethics Code states: “If a psychologist finds that, due to unforeseen factors, a potentially harmful multiple relationship has arisen, the psychologist takes reasonable steps to resolve it with due regard for the best interests of the affected person and maximal compliance with the Ethics Code.” It may become necessary for Dr. R. to refer the client to another therapist or to end the relationship with the client’s sister, but neither of these would be the best initial course of action.
d. Incorrect See explanation for response c.

The correct answer is: discuss the matter with the client as soon as possible to help determine the best course of action.

106
Q

An employee who complains that personnel decisions at the company where he works are inconsistent, biased, and unethical is expressing concern about which type of justice?
Select one:

A.
procedural

B.
distributive

C.
retributive

D.
interactional

A

Experts interested in organizational justice distinguish between three types - procedural, distributive, and interactional.
a. CORRECT Procedural justice refers to the fairness of the way in which policies and procedures are implemented. According to Leventhal et al. (1980), there are six rules of procedural justice: consistency, bias suppression, accuracy, correctability, representativeness, and ethicality. [G. S. Leventhal, J. Karuza, and W. R. Fry, Beyond fairness: A theory of allocation preferences, in G. Mikula (Ed.), Justice and social interaction, New York, Springer-Verlag, 1980.]

b. Incorrect Distributive justice refers to the fairness of salaries, benefits, and other outcomes.
c. Incorrect Retributive justice is not included as a type of organizational justice and refers to rectifying an injustice - e.g., ensuring that people who commit illegal or immoral acts get the punishment they deserve.
d. Incorrect Interactional justice refers to perceptions regarding the quality and content of person-to-person interactions and includes an employee’s perceptions regarding employee-employer interactions during hiring, termination, and other procedures.

The correct answer is: procedural

107
Q

Your client is Angie A. who has been seeing you for a long-standing gambling problem. She is not responding to the treatment you have been providing and you decide to consult with a colleague who has extensive experience working with clients who have a gambling addiction. Before doing so:
Select one:

A.
you must obtain consent from Angie.

B.
you must obtain consent from Angie unless this is an emergency situation.

C.
you must obtain consent from Angie unless you will be able to disguise her identity when talking to the consultant.

D.
you do not need to obtain consent from Angie as long as you discuss only information relevant to the purpose of the consultation with the consultant.

A

This issue is addressed in Standard 4.06 of the APA’s Ethics Code and Principle I.45 of the Canadian Code of Ethics for Psychologists.
a. Incorrect See explanation for response c.

b. Incorrect See explanation for response c.
c. CORRECT This answer is most consistent with ethical requirements. For example, Standard 4.06 states that, when consulting with a colleague, “psychologists do not disclose confidential information that could reasonably lead to the identification of a client/patient … umless they have obtained the prior consent of the person … or the dislosure cannot be avoided.”
d. Incorrect See explanation for response c.

The correct answer is: you must obtain consent from Angie unless you will be able to disguise her identity when talking to the consultant.

108
Q

A manager or consultant interested in maximizing her influence on employees should be aware that the productivity of employees is most likely to be positively affected if she relies on which bases of social power?
Select one:

A.
expert and informational

B.
informational and legitimate

C.
legitimate and referent

D.
expert and referent

A

French and Raven (1959) distinguished between six bases of social power: reward, coercive, legitimate, referent, informational, and expert.
a. Incorrect Expert power is important for a manager or consultant (see response d), but the problem with informational power is that, once the employee has the information, the manager or consultant would no longer have power.

b. Incorrect A person has legitimate power when he/she is accepted as the “boss” and informational power when he/she has needed information. Over the long-run, neither of these sources of power would be the most effective.
c. Incorrect Referent power is important (see response d), but legitimate power is not associated with increased productivity.
d. CORRECT Several authorities have suggested that incremental power (which is a combination of expert and referent power) is optimal for managers and consultants because it has the greatest positive effects on employee performance. A person has expert power when he/she has needed expertise and referent power when he/she is liked and respected.

The correct answer is: expert and referent

109
Q

For nearly two years, a 30-year old woman has had several physical complaints that apparently do not have a physical cause. Her symptoms have changed over time and have included headaches and backaches, joint pain, and weakness in her hands and fingers. She says that she spends a lot of time worrying about her health and that her symptoms have made her miss work and that she’s afraid she’s going to get fired. This woman’s symptoms are most suggestive of:
Select one:

A.
Factitious Disorder.

B.
Somatic Symptom Disorder.

C.
Generalized Anxiety Disorder.

D.
Conversion Disorder.

A

Answer B is correct: The essential feature of Somatic Symptom Disorder is the presence of one or more somatic symptoms that cause distress or a significant disruption in daily life accompanied by excessive thoughts, feelings, or behaviors related to the symptoms.

Answers A, C, and D: See explanation for answer B.

The correct answer is: Somatic Symptom Disorder.

110
Q

The primary purpose of “informed consent” is best defined as:
Select one:

A.
providing a client with the information needed to make a reasonable decision about the services offered.

B.
providing a client with information about the particular situations in which confidentiality may be breached.

C.
ensuring that a client is provided with appropriate and adequate services.

D.
protecting the practitioner from malpractice claims.

A

An informed consent should be obtained prior to providing psychological services to an individual and prior to a subject’s participation in research.
a. CORRECT This question is pretty straightforward. The goal of an informed consent is to protect the recipient of psychological services (or subjects in research studies) by providing them with the information they need to make an informed decision about whether or not to participate.

b. Incorrect Although this is one of the functions of an informed consent, this is not as good a description as response a.
c. Incorrect See explanation for response a.
d. Incorrect See explanation for response a.

The correct answer is: providing a client with the information needed to make a reasonable decision about the services offered.

111
Q

The Stone Center’s self-in-relation theory attributes a woman’s core sense of self to which of the following?
Select one:

A.
the development of a mutually empathic bond with her mother during early childhood

B.
her ability to resolve the conflicting demands of the id, ego, and superego during childhood

C.
her ability to separate “self” and “object” during the separation-individuation process

D.
the development of a unique “autonomous” self during adolescence

A

Self-in-relation theory was originally developed at the Stone Center at Wellesley College by several feminist psychologists and researchers.
a. CORRECT Self-in-relation theory proposes that a woman’s core self-structure is an interacting or relational self that emerges from the early mother-daughter relationship and the development of mutual empathy in that relationship.

b. Incorrect See explanation above.
c. Incorrect See explanation above.
d. Incorrect See explanation above.

The correct answer is: the development of a mutually empathic bond with her mother during early childhood

112
Q

The development of the behaviorally anchored rating scale (BARS) was based on which of the following?
Select one:

A.
Gagne’s (1965) taxonomy

B.
Flanagan’s (1963) critical incidents

C.
Blanz and Ghiselli’s (1972) performance dimensions

D.
Kirkpatrick’s (1976) evaluation criteria

A

Knowing that the “behavioral anchors” used in BARS are critical incidents would have helped you identify the correct answer to this question.
a. Incorrect See explanation for response b.

b. CORRECT Smith and Kendall (1963) incorporated critical incidents into BARS in order to improve the accuracy of performance ratings.
c. Incorrect See explanation for response b.
d. Incorrect See explanation for response b.

The correct answer is: Flanagan’s (1963) critical incidents

113
Q

Loss of licensure was the most frequent reason for cases opened by the APA’s Ethics Committee from 2000 to 2008. The second-most frequent reason was:
Select one:

A.
a problem related to insurance or fees.

B.
violation of confidentiality.

C.
sexual misconduct with an adult.

D.
a nonsexual dual relationship.

A

Data on cases opened by the Ethics Committee from 2000 to 2008 are summarized by K. S. Pope and M. J. T. Vasquez in Ethics in psychotherapy and counseling: A practical guide, Hoboken, NJ, John Wiley & Sons, 2010.
a. Incorrect A problem related to insurance or fees was the primary factor in none of the opened cases and one of multiple factors in 13% of cases.

b. Incorrect A violation of confidentiality was the primary factor in 1% of opened cases and one of multiple factors in 4% of cases.
c. CORRECT Sexual misconduct with an adult was the primary factor in 4% of cases and one of multiple factors in 33% of cases.
d. Incorrect A nonsexual dual relationship was the primary factor in 3% of cases and one of multiple factors in 14% of cases.

The correct answer is: sexual misconduct with an adult.

114
Q

A person with associative visual agnosia:
Select one:

A.
can name a familiar object she sees but does not know how to use it.

B.
can name a familiar object when she does so spontaneously but not when she is asked to do so.

C.
cannot name a familiar object she sees but may recognize it when it is placed in her hand and cannot copy or match a drawing of the object.

D.
cannot name a familiar object she sees but may know what it is used for and can copy or match a drawing of the object.

A

The term visual agnosia refers to the inability to recognize familiar objects by sight. Associative and apperceptive visual agnosia are two types of visual agnosia that you want to be familiar with for the EPPP.
a. Incorrect This is not characteristic of visual agnosia.

b. incorrect This does not describe associative visual agnosia.
c. Incorrect This response describes the condition known as apperceptive visual agnosia.
d. CORRECT This response describes associative visual agnosia, which occurs when visual and language areas become disconnected.

The correct answer is: cannot name a familiar object she sees but may know what it is used for and can copy or match a drawing of the object.

115
Q

A veteran of the Iraq war complains about tremor in his right hand and weakness in his left arm. He has been examined by several doctors who have been unable to find a cause for his symptoms, and the last doctor he saw told him that his symptoms aren’t consistent with any known neurological disease. The man’s wife tells you that he accidentally shot and seriously wounded a fellow soldier while in Iraq but that he has no memory of doing so. The most likely diagnosis for this man is:
Select one:

A.
Factitious Disorder.

B.
PTSD.

C.
Malingering.

D.
Conversion Disorder.

A

Answer D is correct: Conversion Disorder involves a disturbance in voluntary motor or sensory functioning that suggests a serious neurological or other medical condition with evidence of an incompatibility between the symptom and recognized neurological and medical conditions.

Answer A: Factitious Disorder can be ruled out because there is no indication that the man is falsifying his symptoms or that his symptoms are associated with some type of deception.

Answer B: The man is not exhibiting any of the characteristic symptoms of PTSD.

Answer C: Malingering is characterized by the intentional production of physical or psychological symptoms for the purpose of obtaining an external reward.

The correct answer is: Conversion Disorder.

116
Q

In conflict resolution, the primary role of a mediator is to:
Select one:

A.
derive a workable solution to the problem that will be acceptable to all disputants.

B.
have disputants offer solutions and then choose the best one him/herself.

C.
help disputants consider alternative solutions to the problem.

D.
act as a “go-between” by working with each disputant individually.

A

Mediators facilitate the conflict resolution process. While they may offer suggestions, they do not determine the solution to a problem.
a. Incorrect See explanation for response c.

b. Incorrect See explanation for response c.
c. CORRECT Disputants are often inflexible with regard to their own idea of the best solution to a problem, and an important task for a mediator is to increase disputants’ flexibility regarding alternatives. One way to achieve this goal is to suggest alternatives.
d. Incorrect A mediator may do this initially when conflict and hostility are high, but it’s only a temporary role and not a primary one.

The correct answer is: help disputants consider alternative solutions to the problem.

117
Q

Research on the impact of mental practice on athletic performance has found that it is:
Select one:

A.
more beneficial for experienced (skilled) individuals than for novices and when it makes use of internal imagery.

B.
more beneficial for novices than for experienced individuals and when it makes use of external imagery.

C.
equally as beneficial for experienced individuals and novices when it makes use of internal imagery.

D.
equally as effective for experienced individuals and novices regardless of the type of imagery used.

A

Mental practice refers to the use of mental images in the absence of any overt physical activity to learn or improve one’s skills. It is frequently used to enhance athletic performance.
a. CORRECT Studies on mental practice have confirmed that it is useful for improving athletic performance when added to physical practice but that its effects are moderated by the skill level of the individual, the type of imagery, and the type of task. Specifically, the research has shown that mental practice is less effective for novices than for skilled individuals; is more effective when it makes use of internal imagery (i.e., imagining the behavior as though one is actually performing it rather than observing oneself performing it); and is more effective for sports that have cognitive elements (e.g., dance and gymnastics) than for sports that rely primarily on strength or motor skills.

b. Incorrect See explanation for response a.
c. Incorrect See explanation for response a.
d. Incorrect See explanation for response a.

The correct answer is: more beneficial for experienced (skilled) individuals than for novices and when it makes use of internal imagery.

118
Q

Kohlberg originally identified the six stages of moral development while conducting research for his doctoral dissertation. That research involved asking 72 white male children to respond to a dilemma involving:
Select one:

A.
a man named Heinz who stole a drug for his dying wife.

B.
a man named Albert who stole a car but didn’t know the difference between right and wrong.

C.
a boy named Hans who mistreated his horse.

D.
a boy named Alfred who lied in order to protect his younger brother.

A

Kohlberg originally described his stages of moral development in his doctoral dissertation.
a. CORRECT Kohlberg’s approach to studying moral development involves investigating how the individual responds to a moral dilemma. The first dilemma that he developed (and used in his dissertation) was the Heinz dilemma, which involved a poor man who needed a drug for his dying wife.

b. Incorrect See explanation for response a.
c. Incorrect See explanation for response a.
d. Incorrect See explanation for response a.

The correct answer is: a man named Heinz who stole a drug for his dying wife.

119
Q

A new job selection test with a validity coefficient of .30 will most likely improve decision-making accuracy when:
Select one:

A.
the job applicant pool is very large.

B.
the job applicant pool is very small.

C.
the number of successful employees hired without the new test is very large.

D.
the number of successful employees hired without the new test is very small.

A

To answer this question, you have to know that a predictor’s incremental validity is affected by the selection ratio and the base rate.
a. CORRECT A predictor is most likely to increase decision-making accuracy when there are many applicants for each job (i.e., when there is a low selection ratio).

b. Incorrect See explanation for response a.
c. Incorrect A measure is most likely to have incremental validity when the base rate is moderate (versus low or high).
d. Incorrect See explanations for responses a and c.

The correct answer is: the job applicant pool is very large.

120
Q

Stepwise multiple regression would be most useful for:
Select one:

A.
identifying the smallest number of predictors needed to accurately predict scores on a criterion.

B.
comparing the strength of the relationship between pairs of predictors and a single criterion.

C.
determining which predictors have a causal effect on two or more criteria.

D.
determining the combination of criteria that are most accurately predicted by a set of predictors.

A

Multiple regression is used when the goal is to use several predictors to predict or estimate performance on a single criterion.
a. CORRECT When using stepwise multiple regression, predictors are added or deleted one at a time to determine the minimum number necessary to accurately predict or estimate performance on the criterion.

b. Incorrect See explanation above.
c. Incorrect See explanation above.
d. Incorrect See explanation above.

The correct answer is: identifying the smallest number of predictors needed to accurately predict scores on a criterion.

121
Q

The failure of __________ to produce a conditioned response provides evidence that the effectiveness of classical conditioning is not due to temporal contiguity.
Select one:

A.
two-factor learning

B.
backward conditioning

C.
associative learning

D.
second-order conditioning

A

The effectiveness of classical conditioning depends on several factors including the temporal relationship between presentation of the CS and US.
a. Incorrect See explanation for response b.

b. CORRECT Backward conditioning involves presenting the US prior to the CS and usually does not produce a conditioned response, and its ineffectiveness is used as evidence that contingency (rather than contiguity) underlies classical conditioning.
c. Incorrect See explanation for response b.
d. Incorrect See explanation for response b.

The correct answer is: backward conditioning

122
Q

As described by Piaget, which of the following is a primary concern for individuals in the autonomous stage of moral development?
Select one:

A.
abstract universal principles

B.
socially agreed-upon principles

C.
the need to maintain social rules and laws

D.
the positive and negative consequences of an act

A

Piaget described moral development in terms of two stages – heteronomous and autonomous.
a. Incorrect Universal moral principles are of concern for individuals in Stage 6 of Kohlberg’s stages of moral development.

b. CORRECT Children in Piaget’s autonomous stage view rules as being socially agreed-upon and alterable when people who are governed by them agree to the change.
c. Incorrect The need to maintain social rules and laws is characteristic of Kohlberg’s Stage 4.
d. Incorrect Children in Piaget’s heteronomous stage (and Kohlberg’s Stages 1 and 2) focus on the consequences of an act.

The correct answer is: socially agreed-upon principles

123
Q

A researcher interested in Chodorow’s (1989) object relations approach to feminist theory would be most likely to conduct a study to:
Select one:

A.
assess the impact of birth order on women’s personality in adulthood.

B.
compare the stability of self-esteem in girls and boys from age 10 to age 18.

C.
assess the impact of father absence in early childhood on women’s early romantic/sexual relationships.

D.
evaluate the impact of maternal absence in infancy on women’s social relations in early adulthood.

A

Chodorow reformulated traditional object relations theory by proposing that women (who continue to be the primary caregivers of young children) interact with male and female children differently, which contributes to gender differences in personality and social relatedness.
a. Incorrect See explanation for response d.

b. Incorrect See explanation for response d.
c. Incorrect See explanation for response d.
d. CORRECT Of the studies listed, only this one addresses the impact of the early mother-daughter relationship on the daughter’s future social relations (which is a focus of Chodorow’s theory).

The correct answer is: evaluate the impact of maternal absence in infancy on women’s social relations in early adulthood.

124
Q

From about ________ of age, children use telegraphic speech (two-word utterances) to communicate an entire sentence.
Select one:

A.
8 to 12 months

B.
12 to 18 months

C.
18 to 24 months

D.
24 to 30 months

A

Telegraphic speech consists of two words that convey the meaning of entire sentences.
a. Incorrect See explanation for response c.

b. Incorrect See explanation for response c.
c. CORRECT Children exhibit a spurt in vocabulary growth between 18 and 24 months and combine two words to construct a short sentence.
d. Incorrect See explanation for response c.

The correct answer is: 18 to 24 months

125
Q

With regard to informed consent, what are your ethical obligations when performing a child custody evaluation that will entail evaluating all involved parties?
Select one:

A.
You are required to obtain consent from each parent (or other participating adult) prior to conducting the evaluation.

B.
You are required to obtain consent from the attorney of each parent (or other participating adult) prior to conducting the evaluation.

C.
You are required to attempt to obtain consent from each parent (or other participating adult) prior to releasing the results of the evaluation to the court.

D.
You are not required to obtain the consent of the parents (or other participating adults) prior to conducting the evaluation.

A

Psychologists and other mental health professionals are ethically obligated to obtain informed consent for all professional forensic activities (M. J. Ackerman, Essentials of Forensic Psychological Assessment, New York, John Wiley & Sons, 1999, p. 12).
a. CORRECT This response is most consistent with ethical requirements – see, e.g., Paragraph 9 of APA’s Guidelines for Child Custody Evaluations in Family Law Proceedings. You would also want to obtain the assent from the child and consent from the child’s legal representative. However, this is not included in any of the answers, so this answer is the best one.

b. Incorrect See explanation above.
c. Incorrect See explanation above.
d. Incorrect See explanation above.

The correct answer is: You are required to obtain consent from each parent (or other participating adult) prior to conducting the evaluation.

126
Q

The technique known as “protocol analysis” was derived from:
Select one:

A.
cognitive science.

B.
Gestalt psychology.

C.
the medical model.

D.
social-learning theory.

A

Knowing that protocol analysis is a qualitative evaluation techique used to assess human thought (cognitive) processes would have helped you identify the correct response to this question.
a. CORRECT The term “cognitive science” refers to a broad range of disciplines and approaches (e.g., cognitive psychology, neuroscience, computer science) that focus on the acquisition and use of knowledge. Protocol analysis involves having the individual “think aloud” while solving a cognitive problem.

b. Incorrect See explanation for response a.
c. Incorrect See explanation for response a.
d. Incorrect See explanation for response a.

The correct answer is: cognitive science.

127
Q

You receive a subpoena duces tecum from the court requesting that you appear in court and provide it with information about one of your clients. Your best course of action would be to:
Select one:

A.
refuse to appear in court or provide information without a consent from the client or the client’s attorney.

B.
appear in court and provide the requested information even if the client has not given consent for you to do so.

C.
appear in court but provide the requested information only if the client’s situation represents a legal exception to privilege.

D.
appear in court but assert the privilege and provide the requested information only after obtaining the client’s consent or a court order.

A

When answering questions related to confidentiality or privilege, the best answer is usually the most conservative one.
a. Incorrect See explanation for response d.

b. Incorrect See explanation for response d.
c. Incorrect See explanation for response d.
d. CORRECT A psychologist’s first response to a subpoena duces tecum should be to seek legal advice to determine what his/her responsibilities are. Since this is not one of the choices, this is the best answer. In most cases, the most appropriate action is to appear in court as requested but assert the privilege and not release information about a client unless the client has authorized such release or the court has ordered the release.

The correct answer is: appear in court but assert the privilege and provide the requested information only after obtaining the client’s consent or a court order.

128
Q

To determine the degree of association between number of hours studied and score on the EPPP when data on both variables have been converted to ranks, you would use which of the following?
Select one:

A.
Spearman rho

B.
Pearson r

C.
point biserial

D.
phi coefficient

A

In this situation, you want to determine the correlation between two variables that are reported in terms of ranks.
a. CORRECT The Spearman rho (also known as the Spearman rank-order correlation coefficient) is used to determinee the degree of association between two variables that are expressed as ranks.

b. Incorrect The Pearson r is used when both variables are continuous.
c. Incorrect The point biserial is the appropriate correlation coefficient when one variable is continuous and the other is a true dichotomy.
d. Incorrect The phi coefficient is used when both variables are true dichotomies.

The correct answer is: Spearman rho

129
Q

The tendency for interrupted and unfinished tasks to be remembered better than completed tasks when the tasks are performed under non-stressful conditions is referred to as the:
Select one:

A.
Zeigarnik effect.

B.
Hawthorne effect.

C.
Rosenthal effect.

D.
von Restorff effect.

A

This question describes the Zeigarnik effect. Note that the opposite may occur when tasks are performed under stressful (versus non-stressful) conditions.
a. CORRECT The Zeigarnik effect is associated with Lewin’s field theory, which views behavior as a function of the interaction between the person and the environment.

b. Incorrect The Hawthorne effect refers to the tendency of research participants to behave differently simply because they are participating in a research study.
c. Incorrect The Rosenthal effect is also known as the “self-fulfilling prophecy” effect and is not relevant to the phenomenon described in this question.
d. Incorrect The von Restorff effect refers to the tendency to pay attention to or recall stimuli that are distinct from other stimuli (e.g., a noun in a list of verbs).

The correct answer is: Zeigarnik effect.

130
Q

To be consistent with the Federal Educational Rights and Privacy Act (FERPA), a school psychologist must keep in mind that:
Select one:

A.
it is necessary to obtain the signed consent of a student’s parents before releasing any information from the student’s file to officials of another school where the student is planning to enroll.

B.
student records, including identifying information, must be released to designated federal and state educational authorities if needed in connection with the evaluation of federally sponsored educational programs.

C.
parents have the right not only to inspect and review their children’s school records but also to challenge the contents of the records.

D.
all of the above.

A

For the exam, you want to be familiar with the requirements of FERPA (which is also known as the Buckley Amendment). Additional information on FERPA is provided in the chapter on ethics and professional issues in the written study materials.
a. Incorrect Written consent is NOT needed in this situation, as long as the parents have been notified of the transfer of records to the new school.

b. Incorrect Although records must be supplied in this situation, identifying information must be removed from the records.
c. CORRECT This is a requirement of FERPA.
d. Incorrect Only response c is correct.

The correct answer is: parents have the right not only to inspect and review their children’s school records but also to challenge the contents of the records.

131
Q

When treating an 8-year old child who doesn’t like going to bed at night because he’s afraid of the dark, which of the following interventions would probably be most successful?
Select one:

A.
a social skills technique that relies on modeling with guided participation and behavioral rehearsal

B.
a cognitive self-control technique that incorporates visual imagery and positive self-statements

C.
a behavioral technique that includes flooding (in vivo exposure with response prevention)

D.
a behavioral technique that includes systematic desensitization

A

In general, behavioral and cognitive-behavioral techniques have been found most effective for alleviating phobias in both children and adults. Unfortunately, knowing this doesn’t help with this question since all of the techniques listed fall into this category. Considering the boy’s age may have helped you eliminate at least two of the answers (c and d).
a. Incorrect Participant modeling has been found to be more useful for childhood phobias involving animals and dental and medical procedures.

b. CORRECT This technique has been described by A. M. Graziano and K. C. Mooney (Children and Behavior Therapy, New York, Aldine, 1984), who found it to be an effective treatment for children who fear the dark. Cognitive self-control involves several steps: At bedtime, the child first relaxes, then visualizes a pleasant scene, and then makes self-statements such as “I am brave. I can take care of myself in the dark.” This technique is considered a self-control technique since it is administered by the child him/herself (although the parents are also involved in reminding the child to use the technique and in monitoring the child’s progress).
c. Incorrect The use of flooding with children is controversial and is not considered a treatment-of-choice by most authorities.
d. Incorrect Systematic desensitization has generally not been found to be particularly effective with children (see Graziano and Mooney for a review of the literature).

The correct answer is: a cognitive self-control technique that incorporates visual imagery and positive self-statements

132
Q

Gary G. has written an article based on his doctoral dissertation. He would like to list his dissertation chairperson as the first author when he submits the article for publication because the chairperson is very well known in the field and doing so will increase the likelihood that the article will be published. This is:
Select one:

A.
acceptable if the research for the dissertation was derived from the chairperson’s research.

B.
acceptable as long as Gary notes in a footnote that the study was his dissertation research project.

C.
acceptable since Gary is the person who has decided to list the chairperson as first author.

D.
not acceptable.

A

Publication credit is addressed in Standard 8.12 of the Ethics Code and Principle III.7 of the Canadian Code of Ethics for Psychologists.
a. Incorrect See explanation for response d.

b. Incorrect See explanation for response d.
c. Incorrect See explanation for response d.
d. CORRECT Publication credit must reflect the contribution of the individual to the research and, when an article is substantially based on a student’s doctoral dissertation, the student is ordinarily listed as first author.

The correct answer is: not acceptable.

133
Q

Color coding the keys of the piano to help a music student remember and play musical scales would be most helpful if the student has which of the following conditions?
Select one:

A.
anosognosia

B.
achromatopsia

C.
synesthesia

D.
simultanagnosia

A

For the exam, you want to be familiar with all of the conditions listed in the answers to this question. These are described in the Physiological Psychology and Psychopharmacology chapter of the written study materials.
a. Incorrect See explanation for response c.

b. Incorrect See explanation for response c.
c. CORRECT A music student with sound-color synesthesia would be likely to benefit from color-coding the piano keys since he/she “sees” sounds in different colors.
d. Incorrect See explanation for response c.

The correct answer is: synesthesia

134
Q

In terms of drug use, a person is exhibiting “habituation” when he/she:
Select one:

A.
has developed physical dependence on the drug.

B.
requires higher and higher doses of the drug to achieve the same effects.

C.
has a desire to continue using the drug with little or no desire to increase the amount of the drug.

D.
experiences reduced symptoms following cessation of the drug due to multiple withdrawal experiences.

A

Answer C is correct: The meaning of the term “habituation” depends on the context in which it is used. From the perspective of pharmacology, habituation occurs when repeated use of a drug results in a desire for continued use with little or no desire to increase the amount or dose of the drug and no physical dependence on the drug.

Answers A, B, and D: See explanation for answer C.

The correct answer is: has a desire to continue using the drug with little or no desire to increase the amount of the drug.

135
Q

A researcher is most likely to choose the multiple-baseline design for her research study because:
Select one:

A.
she wants to eliminate the confounding effects of “demand characteristics.”

B.
she wants to eliminate the confounding effects of repeated measurement of the dependent variable.

C.
she wants to avoid having to remove an effective treatment during the course of the study.

D.
she wants to determine if “nonspecific factors” are responsible for any observed effects of the treatment.

A

Use of the multiple baseline design involves sequentially applying a treatment across two or more “baselines” - i.e., across two or more participants, settings, or behaviors.
a. Incorrect See explanation for response c.

b. Incorrect See explanation for response c.
c. CORRECT One advantage of the multiple baseline design is that its use does not require removing a treatment once it has been applied to a baseline for the duration of the study.
d. Incorrect See explanation for response c.

The correct answer is: she wants to avoid having to remove an effective treatment during the course of the study.

136
Q

A practitioner of Beck’s cognitive-behavioral therapy adopts the role of ___________ when working with therapy clients.
Select one:

A.
supervisor

B.
teacher

C.
collaborator

D.
consultant

A

A key characteristic of Beck’s CBT is its emphasis on the active participation of the client.
a. Incorrect See explanation for response c.

b. Incorrect See explanation for response c.
c. CORRECT Beck refers to his therapeutic approach as “collaborative empiricism,” in part, because the client is encouraged to be actively engaged in the therapeutic process - i.e., to collaborate with the therapist in defining goals, evaluating the progress of therapy, and so on.
d. Incorrect See explanation for response c.

The correct answer is: collaborator

137
Q

Research on the effects of the SSRI fluoxetine for individuals with Bulimia Nervosa has found that:
Select one:

A.
it is useful only for reducing comorbid symptoms of depression.

B.
it is useful for reducing symptoms of Bulimia and comorbid symptoms of depression.

C.
it is more effective than cognitive-behavioral therapy for reducing symptoms of Bulimia.

D.
it has a paradoxical effect that reduces the likelihood of a positive response to cognitive-behavioral therapy.

A

Answer B is correct: Several studies have found that the antidepressant fluoxetine is effective for alleviating the symptoms of comorbid depression and for reducing the symptoms of Bulimia regardless of the presence or absence of depressive symptoms. However, it has not been found to be more effective than CBT although, for some patients, a combination of CBT and fluoxetine is more effective than CBT alone. See, e.g., D. J. Goldstein et al., Effectiveness of fluoxetine therapy in bulimia nervosa regardless of comorbid depression, International Journal of Eating Disorders, 25(1), 19-27, 1999; A. J. Zhu and B. T. Walsh, Pharmacologic treatment for eating disorders, Canadian Journal of Psychiatry, 47, 227-234, 2002.

Answers A, C, and D: See explanation for answer B.

The correct answer is: it is useful for reducing symptoms of Bulimia and comorbid symptoms of depression.

138
Q

Intercept bias is occurring when scores on a predictor:
Select one:

A.
consistently underpredict or overpredict the criterion performance of a particular group of examinees.

B.
consistently underpredict the criterion performance of all examinees.

C.
are related to the criterion performance of one group of examinees but not of another group.

D.
are unrelated to the criterion performance of examinees in all groups.

A

Intercept bias is another name for unfairness, which is identified in the EEOC’s Uniform Guidelines on Employee Selection Procedures as a cause of adverse impact.
a. CORRECT In a scatterplot, the intercept is the point at which the regression line intersects the Y-axis. An intercept bias occurs when the regression lines for different groups of examinees intersect the Y-axis at different points, which means that the predictor systematically overpredicts or underpredicts the performance of members of a specific group.

b. Incorrect See explanation for response a.
c. Incorrect This answer describes a slope bias which is another name for differential validity and is another potential cause of adverse impact.
d. Incorrect This answer describes a predictor that is invalid for all groups.

The correct answer is: consistently underpredict or overpredict the criterion performance of a particular group of examinees.

139
Q

Based on their review of the research on eye movement desensitization and reprocessing (EMDR), Davidson and Parker (2001) concluded that eye movements:
Select one:

A.
are the most important contributor to the effectiveness of the treatment.

B.
are one of three essential contributors to the effectiveness of the treatment.

C.
are not an essential contributor to the effectiveness of the treatment.

D.
actually decrease the effectiveness of the treatment.

A

The research on EMDR has not produced entirely consistent results. However, Davidson and Parker’s (2001) meta-analysis found that eye movements do not contribute to its effectiveness and that its benefits are due, instead, to exposure to the feared stimulus.
a. Incorrect See explanation for response c.

b. Incorrect See explanation for response c.
c. CORRECT These investigators concluded that the treatment has similar effects with and without the inclusion of eye movements.
d. Incorrect See explanation for response c.

The correct answer is: are not an essential contributor to the effectiveness of the treatment.

140
Q

Motivational interviewing and __________ share the assumption that interventions are most effective when they match the client’s readiness to change.
Select one:

A.
the ABC model

B.
the transtheoretical model

C.
person-centered therapy

D.
solution-focused therapy

A

As its name implies, motivational interviewing was developed for people who are ambivalent about changing their behavior.
a. Incorrect The ABC model underlies Ellis’s REBT and other cognitive-behavioral approaches and views behavior as the result of a sequence involving an activating event (A), beliefs about that event (B), and the consequences of those beliefs (the behavior).

b. CORRECT Knowing that the transtheoretical model is also known as the stages of change model would have helped you identify this as the correct answer.
c. Incorrect Although motivational interviewing incorporates elements of person-centered therapy, person-centered therapy is not based on the assumption that an intervention is most effective when it matches the client’s readiness to change.
d. Incorrect Solution-focued therapy does not explictly address the client’s readiness to change.

The correct answer is: the transtheoretical model

141
Q

To eliminate the least qualified job applicants early in the selection process, which of the following would be most helpful?
Select one:

A.
profile matching

B.
multiple cutoff

C.
critical incident technique

D.
multiple hurdle

A

To answer this question, you need to be familiar with each of the techniques listed to determine which one best fits the task described in the question. For vague questions like this one, always pick the most obvious answer–don’t talk yourself into a less obvious one.
a. Incorrect Profile matching involves administering several selection measures and comparing the applicant’s score profile to that of the typical successful employee.

b. Incorrect The multiple cutoff technique entails administering all selection measures to an applicant and considering the applicant only if he/she scores above a predefined minimum score on each measure.
c. Incorrect The critical incident technique is used in performance appraisals to determine how well an employee is doing and to provide the employee with feedback about his/her performance.
d. CORRECT As its name implies, the multiple hurdle technique involves administering selection measures one at a time, with each successive measure (hurdle) being administered only when the applicant has been successful on the previous one. The first hurdle is designed to eliminate the least qualified applicants.

The correct answer is: multiple hurdle

142
Q

Which of the following is true about the effects of ECT on memory when it is used as a treatment for depression?
Select one:

A.
Memory for autobiographical information is more adversely affected than is memory for impersonal information.

B.
Anterograde amnesia is a more severe and persistent adverse effect of ECT than is retrograde amnesia.

C.
Right unilateral (vs. bilateral) electrode placement can reduce the incidence and severity of retrograde amnesia.

D.
Right unilateral electrode placement reduces anterograde amnesia but produces greater retrograde amnesia than does bilateral placement.

A

Answer C is correct: Although ECT has been found to have good short-term effects on depressive symptoms, its use is considered controversial by some experts because of its relatively high relapse rates and adverse effects on memory. However, research has confirmed that right (non-dominant) unilateral ECT causes less retrograde and anterograde amnesia than does bilateral ECT. See, e.g., H. A. Sackheim et al., A prospective, randomized double-blind comparison of bilateral and right unilateral electroconvulsive therapy at different stimulus levels, Archives of General Psychiatry, 57, 425-434, 2000.

Answer A: This statement is the opposite of what is true. Memory for autobiographical information is affected less than impersonal information.

Answer B: This statement is also the opposite of what is true. Although many patients (especially those receiving bilateral ECT) experience both types of amnesia, retrograde amnesia is more severe and persistent than is anterograde amnesia.

Answer D: See explanation for answer C.

The correct answer is: Right unilateral (vs. bilateral) electrode placement can reduce the incidence and severity of retrograde amnesia.

143
Q

Divergent thinking is associated with which of the following?
Select one:

A.
practical intelligence

B.
creativity

C.
neuroticism

D.
crystallized intelligence

A

Divergent thinking refers to the ability to think flexibly and consider a wide range of solutions.
a. Incorrect Sternberg distinguishes between three types of intelligence - analytic, creative, and practical. Divergent thinking is most associated with creative thinking.

b. CORRECT Creativity involves a number of attributes including divergent thinking, open-mindedness, unconventionality, and willingness to take risks.
c. Incorrect Neuroticism has not been associated with divergent thinking.
d. Incorrect Convergent (rather than divergent) thinking is associated with crystallized intelligence.

The correct answer is: creativity

144
Q

Research on “gendered environments” suggests that, in the classroom:
Select one:

A.
boys have more interactions with teachers and tend to receive more feedback and praise.

B.
girls have more interactions with teachers and tend to receive more feedback and praise.

C.
boys get more praise from teachers, while girls receive more criticism.

D.
boys get more criticism from teachers, while girls receive more praise and assistance.

A

The research on teacher behaviors is inconsistent and difficult to summarize. However early studies as well as more recent ones do seem to suggest that teachers respond differently to male and female students.
a. CORRECT Of the responses given, this is the best summary of the research. Males receive more attention from teachers and that attention often fosters better academic achievement.

b. Incorrect See explanation for response a.
c. Incorrect See explanation for response a.
d. Incorrect See explanation for response a.

The correct answer is: boys have more interactions with teachers and tend to receive more feedback and praise.

145
Q

Dr. Fanny Fleese, a licensed psychologist, regularly waives the co-payment for low-income clients who are covered by insurance. Waiving the co-payment is:
Select one:

A.
ethical only if the insurance company does not explicitly prohibit this policy.

B.
ethical only if the insurance company has approved of this arrangement.

C.
ethical only if Dr. Fleese doesn’t bill the insurance company at a higher rate so that she collects her full fee.

D.
ethical since Dr. Fleese’s policy is in the best interests of her clients.

A

This issue is addressed in Standard 6.06 of the APA’s Ethics Code and Principle III.5 of the Canadian Code of Ethics for Psychologists.
a. Incorrect See explanation for response b.

b. CORRECT Most insurance companies agree to pay a specified percentage of a client’s fee and, when the co-payment is routinely waived, this means that the company is paying the entire fee. Consequently, waiving the co-payment without the permission of the insurance company represents insurance fraud and is both unethical and illegal.
c. Incorrect See explanation for response b.
d. Incorrect See explanation for response b.

The correct answer is: ethical only if the insurance company has approved of this arrangement.

146
Q

Sleeping immediately after studying on the night before your 8:00 a.m. exam would help minimize the effects of which of the following on your ability to recall information during the exam?
Select one:

A.
cue-dependent forgetting

B.
trace decay

C.
retroactive interference

D.
proactive interference

A

The phenomena listed in the answers to this question are used to explain why we are unable to recall information.
a. Incorrect Cue dependent forgetting refers to the inability to recall information due to inadequate retrieval cues.

b. Incorrect Trace decay theory predicts that memory traces fade over time.
c. CORRECT Retroactive interference occurs when recently learned information interferes with the ability to recall previously learned information. Sleeping between learning information and being tested on that information would eliminate retroactive interference.
d. Incorrect Proactive interference occurs when previously learned information interferes with the ability to recall more recently learned information.

The correct answer is: retroactive interference

147
Q

In a normally shaped distribution, the percentile rank equivalent of a z-score of +2.0 is approximately:
Select one:

A.
2.5.

B.
50.0.

C.
84.0.

D.
97.5.

A

The licensing exam often includes a question or two on the areas under the normal curve. If you remember that about 68% of scores fall between +1.0 and -1.0 standard deviations from the mean, that 95% fall between +2.0 and -2.0 standard deviations from the mean, and that 99% fall between +3.0 and -3.0 standard deviations from the mean, you should be able to identify the correct answer these questions.
a. Incorrect A percentile rank of 2.5 is equivalent to a z-score of about -2.0.

b. Incorrect A percentile rank of 50.0 is equivalent to a z-score of 0.
c. Incorrect A percentile rank of 84 is equivalent to a z-score of +1.0.
d. CORRECT In a normal distribution, a percentile rank of 97.5 is equivalent to a z-score of about +2.0. Note that knowing that 50% of scores fall below the mean and that 47.5% of scores fall between the mean and the score that is 2 standard deviations above the mean would have enabled you to identify the correct answer to this question.

The correct answer is: 97.5.

148
Q

According to Bandura’s (1986) social cognitive theory, most complex human behaviors are the result of:
Select one:

A.
latent learning.

B.
generative learning.

C.
vicarious learning.

D.
insight learning.

A

Knowing that Bandura’s social cognitive theory is also known as social learning theory and observational learning would have helped you identify the correct answer to this question.
a. Incorrect Latent learning is associated with Tolman.

b. Incorrect Generative learning is one of several approaches to constructivism.
c. CORRECT In contrast to behavioral explanations that attribute learning to classical or operant conditioning, Bandura proposed that people acquire a behavior by observing another individual perform that behavior – i.e., they learn vicariously.
d. Incorrect Insight learning is associated with Kohler.

The correct answer is: vicarious learning.

149
Q

To reduce or eliminate a child’s tantrums, which of the following would probably be most effective?
Select one:

A.
positive reinforcement

B.
negative reinforcement

C.
positive punishment

D.
negative punishment

A

In this situation, the goal is to eliminate a behavior. Therefore, punishment is the correct choice.
a. Incorrect Positive reinforcement involves applying a stimulus following a behavior in order to increase that behavior.

b. Incorrect Negative reinforcement involves removing a stimulus following a behavior in order to increase the behavior.
c. Incorrect Positive punishment could, of course, be used, but it is considered objectionable for both ethical and practical reasons (it doesn’t work).
d. CORRECT If you have to choose between positive and negative punishment as an intervention, negative punishment (which involves removing a stimulus following a behavior) is generally preferred. Examples of negative punishment include time-out and response cost.

The correct answer is: negative punishment

150
Q

You receive a subpoena duces tecum from the attorney of someone who is suing a former client of yours. Assuming that you do not have a release from the client, you should:
Select one:

A.
appear as requested but assert the privilege on behalf of the client.

B.
appear as requested but only provide information and records you believe are pertinent to the case.

C.
refuse to appear or release records as requested without a court order.

D.
refuse to appear or release records as requested without a release from the client.

A

A subpoena duces tecum requires an individual to appear at a legal proceeding and bring specified records or other docments.
a. CORRECT There are several things you would want to do in this situation including determining if the subpoena is legally valid and, if so, contacting the client to obtain a release. However, these are not given as options in the answers, and this answer is the best one given the circumstances described in the question. Assuming that the subpoena is legally valid, you would have to appear as requested but would assert the privilege on behalf of your client since you have not received his/her permission to provide confidential information. Note that, while a court order would compel you to testify and provide records, you would also be permitted to do so if the client signs a release, which is why answer c is not a better answer.

b. Incorrese See explanation above.
c. Incorrect See explanation above.
d. Incorrect See explanation above.

The correct answer is: appear as requested but assert the privilege on behalf of the client.

151
Q

Research on blended families has found that:
Select one:

A.
preadolescent girls have less trouble adjusting to a mother-stepfather arrangement than do preadolescent boys.

B.
preadolescent boys have less trouble adjusting to a mother-stepfather arrangement than do preadolescent girls.

C.
preadolescent girls have less trouble adjusting to a mother-stepfather arrangement than to a father-stepmother arrangement.

D.
preadolescent boys and preadolescent girls have less trouble adjusting to a stepparent than do younger children and early adolescents.

A

The mother-stepfather family is the most common type of blended family.
a. Incorrect See explanation for response b.

b. CORRECT Not surprisingly, boys usually adjust better to a stepfather than do girls, and there is evidence that that having a stepfather may actually be beneficial for boys, especially preadolescent boys: Over time, these boys often develop close relationships with their stepfathers and become fairly indistinguishable from boys in nondivorced families in terms of behavioral problems and academic achievement. See, e.g., E. M. Hetherington, An overview of the Virginia Longitudinal Study of Divorce and Remarriage: A focus on early adolescence, Journal of Family Psychology, 7, 39-56, 1993.
c. Incorrect This is the opposite of what is true. Girls (especially preadolescent girls) often have more trouble adjusting to a stepfather than to a stepmother.
d. Incorrect This is also the opposite of what is true. Overall, preadolescent boys and girls typically have more trouble adjusting to the remarriage of a parent than do younger children and early adolescents.

The correct answer is: preadolescent boys have less trouble adjusting to a mother-stepfather arrangement than do preadolescent girls.

152
Q

For _____ of individuals with Parkinson’s disease who are depressed, the depression preceded the onset of motor and other physical symptoms.
Select one:

A.
less than 1%

B.
about 20%

C.
about 50%

D.
at least 75%

A

According to the National Parkinson Foundation (NPF), about 40% of patients with Parkinson’s disease are depressed. In some cases, the depression is endogenous (due to the disease process itself); in others, it is a reaction to the diagnosis and its symptoms.
a. Incorrect See explanation for response b.

b. CORRECT This is the figure cited by the NPF and others and provides evidence that, in some cases, depression related to Parkinson’s disease is not just a reaction to the diagnosis.
c. Incorrect See explanation for response b.
d. Incorrect See explanation for response b.

The correct answer is: about 20%

153
Q

Which of the following would most likely maximize the magnitude of a test’s reliability coefficient?
Select one:

A.
increase the length of the test and increase the homogeneity of the examinees with regard to the attribute measured by the test

B.
decrease the length of the test and increase the homogeneity of the examinees with regard to the attribute measured by the test

C.
increase the length of the test and increase the heterogeneity of the examinees with regard to the attribute measured by the test

D.
decrease the length of the test and increase the heterogeneity of the examinees with regard to the attribute measured by the test

A

Answer C is correct. A test’s reliability coefficient is affected by several factors including its length and the range of test scores. Longer tests tend to be more reliable (assuming that the added items are similar in terms of quality and content to the original items). In addition, the reliability coefficient (like all correlation coefficients) is larger when the range of scores is unrestricted, which occurs when examinees are heterogeneous with regard to the attribute(s) being measured by the test.

The correct answer is: increase the length of the test and increase the heterogeneity of the examinees with regard to the attribute measured by the test

154
Q

The correction for attenuation formula is useful for estimating the magnitude of a criterion measure’s validity coefficient if:
Select one:

A.
all sources of systematic error are controlled.

B.
criterion contamination is eliminated.

C.
measurement error is removed from the predictor and criterion.

D.
the range of criterion scores is unrestricted.

A

The correction for attenuation formula is used to estimate a test’s validity coefficient when the reliability coefficient for the predictor and/or criterion has been increased to 1.0.
a. Incorrect See explanation for response c.

b. Incorrect See explanation for response c.
c. CORRECT Increasing the reliability coefficient to 1.0 means that measurement error has been entirely removed. The correction for attenuation formula, therefore, estimates the validity coefficient when measurement error in the predictor and/or the criterion has been eliminated.
d. Incorrect See explanation for response c.

The correct answer is: measurement error is removed from the predictor and criterion.

155
Q

Which of the following is true about efficacy research?
Select one:

A.
It is most useful for comparing the relative costs and benefits of a treatment.

B.
It is most useful for conducting a summative (versus formative) evaluation of a treatment program.

C.
It is most useful for evaluating a treatment’s effectiveness under controlled conditions.

D.
It is most useful for determining the degree to which a treatment’s effects generalize to other situations and populations.

A

For the exam, you want to know the difference between efficacy and effectiveness research.
a. Incorrect See explanation for response c.

b. Incorrect See explanation for response c.
c. CORRECT Efficacy research is conducted under controlled conditions and, consequently, has good internal validity but limited external validity – i.e., it is useful for establishing whether or not a treatment is effective under controlled conditions but not for evaluating its generalizability to other conditions.
d. Incorrect Effectiveness research uses a correlational or quasi-experimental research design and is useful for evaluating a treatment’s generalizability and cost-effectiveness.

The correct answer is: It is most useful for evaluating a treatment’s effectiveness under controlled conditions.

156
Q

Elevated scores on the MMPI-2’s L and K scales are most indicative of:
Select one:

A.
an attempt to “fake good.”

B.
an attempt to “fake bad.”

C.
random responding.

D.
responding “true” to all items.

A

An examinee’s scores on the MMPI-2’s validity scales provide information on his/her test-taking attitudes.
a. CORRECT An abnormally high score on the L and K scales (usually accompanied by a slightly lower-than-normal score on the F scale) indicates an attempt to present oneself in a favorable light – i.e., to “fake good.”

b. Incorrect See explanation above.
c. Incorrect See explanation above.
d. Incorrect See explanation above.

The correct answer is: an attempt to “fake good.”

157
Q

Which of the following therapies is characterized by the dual goals of helping clients take responsibility for and experience the natural consequences of their actions?
Select one:

A.
transactional analysis

B.
psychodynamic psychotherapy

C.
personal construct therapy

D.
reality therapy

A

The term “responsibility” is a key one in this question and may have helped you identify the correct response.
a. Incorrect See explanation for response d.

b. Incorrect See explanation for response d.
c. Incorrect See explanation for response d.
d. CORRECT According to Glasser, a “success identity” results when one takes responsibility for fulfilling one’s own needs and does so in ways that do not interfere with the ability of others to do the same thing.

The correct answer is: reality therapy

158
Q

Which of the following best describes ethical guidelines regarding a psychologist’s use of client testimonials in advertisements?
Select one:

A.
The use of testimonials is prohibited under any conditions.

B.
The use of testimonials is prohibited only when they are from current clients.

C.
The use of testimonials is prohibited when they are from current clients or, in certain circumstances, former clients.

D.
The use of testimonials is not prohibited as long as they provide accurate information.

A

Testonials are directly addressed in Standard 5.05 of the APA’s Ethics Code and indirectly addressed in Principle III.31 of the Canadian Code of Ethics.
a. Incorrect See explanation for response c.

b. Incorrect See explanation for response c.
c. CORRECT Standard 5.05 states that “psychologists do not solicit testimonials from current therapy clients/patients or other persons who, because of their particular circumstances, are vulnerable to undue influence.”
d. Incorrect See explanation for response c.

The correct answer is: The use of testimonials is prohibited when they are from current clients or, in certain circumstances, former clients.

159
Q

Tokens in a token economy act as which of the following?
Select one:

A.
primary reinforcers

B.
back-up reinforcers

C.
generalized conditioned reinforcers

D.
generalized unconditioned reinforcers

A

In a token economy, tokens have value because they can be exchanged for a variety of unconditioned (primary) reinforcers.
a. Incorrect Primary reinforcers are inherently reinforcing and include food, water, and desirable activities.

b. Incorrect In a token economy, the back-up reinforcers are the primary reinforcers that can be “purchased” with tokens.
c. CORRECT Tokens are generalized conditioned (secondary) reinforcers that can be exchanged for a variety of unconditioned (primary) reinforcers. Like other conditioned reinforcers, they are reinforcing only because they are associated with primary reinforcers.
d. Incorrect See explanation for response c.

The correct answer is: generalized conditioned reinforcers

160
Q

When an action potential occurs, a state of:
Select one:

A.
depolarization is created as positively charged ions enter the cell.

B.
depolarization is created as negatively charged ions enter the cell.

C.
polarization is created as positively charged ions enter the cell.

D.
polarization is created as negatively charged ions enter the cell.

A

An action potential is the electrical impulse that travels though a neuron.
a. CORRECT An action potential is generated by the movement of positively charged sodium ions into the cell, which creates a state of depolarization (i.e., the interior of the cell becomes less negative).

b. Incorrect See explanation above.
c. Incorrect See explanation above.
d. Incorrect See explanation above.

The correct answer is: depolarization is created as positively charged ions enter the cell.

161
Q

At a party, an acquaintance approaches you and asks if you’ve heard the rumor that a colleague of yours is involved in a sexual relationship with a current client of his. The acquaintance says that the rumor seems to be widely known and believed. You think that it’s possible that the rumor is true. What should you do?
Select one:

A.
nothing, since it is only a rumor

B.
suggest to the acquaintance that she discuss the rumor with the colleague

C.
send a letter to the Ethics Committee about the matter

D.
discuss the matter with the colleague yourself

A

As an ethical psychologist, you have a responsibility to follow through on possible ethical violations of colleagues.
a. Incorrect See explanation for response d.

b. Incorrect See explanation for response d.
c. Incorrect See explanation for response d.
d. CORRECT The best course of action would be to confront the colleague directly. (Even if the rumor isn’t true, the colleague should be made aware of it.)

The correct answer is: discuss the matter with the colleague yourself

162
Q

Examinees with Alzheimer’s Dementia would most likely obtain the highest score on which of the following WAIS-IV Indexes?
Select one:

A.
Verbal Comprehension

B.
Working Memory

C.
Perceptual Reasoning

D.
Processing Speed

A

Answer A is correct. The WAIS-IV Technical Manual reports the highest score for individuals with Alzheimer’s Dementia for the Verbal Comprehension Index, followed by the Perceptual Reasoning, Working Memory, and Processing Speed Indexes.

The correct answer is: Verbal Comprehension

163
Q

You are hired by a defense attorney as a consultant in a criminal case. You subsequently receive a subpoena from the prosecuting attorney, requesting that you serve as a fact witness at a deposition. You should:
Select one:

A.
refuse to act as a fact witness because doing so would involve you in a “multiple relationship.”

B.
get permission from the defense attorney before appearing at the deposition.

C.
clarify the expectations of both attorneys with regard to your roles before appearing at the deposition.

D.
appear at the deposition but refuse to provide any information without a court order.

A

In most situations, a psychologist wants to avoid multiple relationships. There are, however, exceptions to this general rule.
a. Incorrect See explanation for response c.

b. Incorrect See explanation for response c.
c. CORRECT This issue is addressed in Standard 3.05(c) of the APA’s Ethics Code and Principle I.26 of the Canadian Code of Ethics. Standard 3.05(c) recommends that psychologists avoid multiple relationships. However, it also states that, when psychologists are “to serve in more than one role in judicial or administrative proceedings, at the outset they clarify role expectations and the extent of confidentiality and thereafter as changes occur.” In other words, it is acceptable to adopt dual roles in some situations. (The acceptability of dual roles in legal proceedings when one role is a fact witness is addressed in the Guidelines for Child Custody Evaluations in Divorce Proceedings.)
d. Incorrect See explanation for response c.

The correct answer is: clarify the expectations of both attorneys with regard to your roles before appearing at the deposition.

164
Q

Your research study includes one independent variable that has four levels and a single dependent variable that is measured on a ratio scale. You decided to analyze your data using a single one-way ANOVA instead of separate t-tests because you want to:
Select one:

A.
statistically analyze both main and interaction effects.

B.
control the experimentwise error rate.

C.
maximize experimental variance.

D.
control the effects of an extraneous variable.

A

An advantage of the analysis of variance is that it makes all possible comparisons simultaneously while holding the probability of making a Type I error at the level of significance set by the researcher.
a. Incorrect A study must include two or more independent variables to analyze main and interaction effects.

b. CORRECT If you used separate t-tests instead of a one-way ANOVA to analyze the data, you would have to conduct multiple comparisons (e.g., a comparison between Level 1 and Level 2, between Level 1 and Level 3, etc.), which would increase the experimentwise error rate.
c. Incorrect You would maximize experimental variance by ensuring that groups are as different as possible with regard to the independent variable.
d. Incorrect The ANCOVA and randomized block ANOVA are used to control the effects of an extraneous variable.

The correct answer is: control the experimentwise error rate.

165
Q

Parham and Helms (1990) developed an attitude measure of racial identity that assesses Cross’s four stages of African American identity development. Parham and Helms’ four stages are:
Select one:

A.
pre-encounter, encounter, resistance-immersion, and introspection.

B.
pre-encounter, encounter, immersion-emersion, and internalization.

C.
contact, disintegration, immersion-emersion, and autonomy.

D.
conformity, dissonance, resistance-immersion, introspection.

A

Answer B is correct. These are the four stages assessed by the Parham and Helms measure. For the exam, you want to be familiar with the four stages identified by Cross as well. These are described in the Clinical Psychology chapter of the written study materials.

The correct answer is: pre-encounter, encounter, immersion-emersion, and internalization.

166
Q

To detect changes in regional cerebral blood flow to facilitate diagnoses of certain brain-impairing conditions, which of the following would be most useful?
Select one:

A.
CT scan and PET

B.
MRI and CT scan

C.
PET and fMRI

D.
CT, MRI, and PET

A

A functional brain imaging technique is used to assess regional cerebral blood flow.
a. Incorrect A CT scan is a structural imaging technique.

b. Incorrect MRI and CT scan are both structural techniques.
c. CORRECT PET (positron emission tomography) and fMRI (functional magnetic resonance imaging) are functional techniques.
d. Incorrect See explanation above.

The correct answer is: PET and fMRI

167
Q
In comparison to White middle-class families, African American middle-class families:
Select one:

A.
are more likely to have a patriarchal structure.

B.
are more likely to have a matriarchal structure.

C.
are less likely to include members of the extended family.

D.
are less likely to exhibit differentiation in power along gender lines.

A
The studies have confirmed that a difference between middle-class African American and White families is that African American families are less rigid with regard to gender roles.
a. Incorrect See explanation for response d.

b. Incorrect See explanation for response d.
c. Incorrect This is the opposite of what is true.
d. CORRECT Although White middle-class families are generally less rigid about gender roles than they were in the past, this is still considered a characteristic that distinguishes African American and White middle-class families.

The correct answer is: are less likely to exhibit differentiation in power along gender lines.

168
Q

A DSM-5 diagnosis of Agoraphobia requires which of the following?
Select one:

A.
the presence of characteristic symptoms in at least two of five designated situations

B.
the duration of symptoms for at least three months

C.
a history of at least one unexpected panic attack

D.
recognition by the person that his/her fear and anxiety are excessive or unreasonable

A

Answer A is correct: The diagnosis of Agoraphobia requires the presence of marked fear or anxiety about at least two of the following situations: using public transportation, being in open spaces, being in enclosed spaces, standing in line or being part of a crowd, and being outside the home alone.

Answer B: The diagnosis of Agoraphobia requires a duration of about six months.

Answers C and D: These are not required for the diagnosis of Agoraphobia.

The correct answer is: the presence of characteristic symptoms in at least two of five designated situations

169
Q

The diagnosis of Opioid Withdrawal requires the development of at least three characteristic symptoms following cessation or reduction of prolonged or heavy use of an opioid or administration of an opioid antagonist after a period of opioid use. Characteristic symptoms include all of the following except:
Select one:

A.
dysphoric mood.

B.
yawning.

C.
muscle aches.

D.
hallucinations or illusions.

A

Answer D is correct: Characteristic symptoms of Opioid Withdrawal include dysphoric mood, nausea or vomiting; muscle aches; lacrimation or rhinorrhea; pupillary dilation, piloerection, or sweating; diarrhea; yawning; fever; and insomnia. Hallucinations and illusions are not characteristic symptoms of this disorder.

Answers A, B, and C: See explanation for answer D.

The correct answer is: hallucinations or illusions.

170
Q

A working mother who has been reassured by her husband and friends many times that she is a “great mother,” states that she is a “terrible mother” after she forgets to wash her daughter’s baseball uniform the night before an important game. Aaron Beck would consider the mother’s conclusion to be an example of which of the following?
Select one:

A.
personalization

B.
arbitrary inference

C.
selective abstraction

D.
dichotomous thinking

A

All of the responses are terms that Beck describes as cognitive distortions.
a. Incorrect Personalization occurs when a person attributes external events to him/herself without evidence supporting such a causal connection.

b. CORRECT Arbitrary inference involves drawing a specific conclusion without supporting evidence or in the face of contradictory evidence.
c. Incorrect Selective abstraction occurs when a person interprets a situation on the basis of a single detail taken out of context (e.g., a jealous husband thinks his wife is having an affair with the neighbor when he sees her pat the neighbor on the back).
d. Incorrect Dichotomous thinking involves categorizing an experience in terms of one of two extremes (e.g., “If I do this, everything will be perfect; if I don’t, everything will be terrible”).

The correct answer is: arbitrary inference

171
Q

The course and extent of recovery from which of the following depends on the cause of the disorder and may involve an acute onset with partial recovery, a stepwise decline, or a progressive course with fluctuations in symptom severity and plateaus that vary in duration?
Select one:

A.
Neurocognitive Disorder due to HIV Infection

B.
Neurocognitive Disorder due to Huntington’s Disease

C.
Vascular Neurocognitive Disorder

D.
Neurocognitive Disorder due to Alzheimer’s Disease

A

Answer C is correct: Vascular Neurocognitive Disorder is caused by cerebrovascular disease. Its course varies, depending on its cause and the location and extent of the damage.

Answers A, B, and D: See explanation for answer C.

The correct answer is: Vascular Neurocognitive Disorder

172
Q

Recovery of cognitive ability following head trauma follows a predictable pattern in most patients. For example, with regard to disorientation, recovery usually occurs in which sequence?
Select one:

A.
orientation to personal information, then to place, and finally to time

B.
orientation to time and place followed by orientation to personal information

C.
orientation to place, then to personal information, and finally to time

D.
orientation to personal information, then to time, and finally to place

A

Recovery of cognitive functions following head trauma usually follows a set pattern. For example, with regard to retrograde amnesia, the most remote memories return first.
a. CORRECT Orientation to personal information is recovered first, followed by place and then time. Patients who continue to be disoriented to personal information rarely exhibit orientation to time or place.

b. Incorrect See explanation for response a.
c. Incorrect See explanation for response a.
d. Incorrect See explanation for response a.

The correct answer is: orientation to personal information, then to place, and finally to time

173
Q

According to Carol Gilligan, adolescent females tend to score at lower stages than adolescent males on Kohlbergian tasks because:
Select one:

A.
there are biases in the way that raters score the responses of females.

B.
females are more concerned than males with being judged positively by the rater.

C.
Kohlberg’s theory focuses on individual rights, while the moral reasoning of girls reflects concerns about responsibility toward others.

D.
Kohlberg’s theory focuses on instrumentality, while the moral responses of girls are based more on socioemotional concerns.

A

Gilligan has criticized Kohlberg’s theory of moral development on the ground that it leaves out a dimension of morality of concern to females.
a. Incorrect See explanation for response c.

b. Incorrect See explanation for response c.
c. CORRECT According to Gilligan, males and females use different moral criteria in judging moral dilemmas, with females emphasizing mutual caring and responsibility for others and males focusing more on individual rights.
d. Incorrect See explanation for response c.

The correct answer is: Kohlberg’s theory focuses on individual rights, while the moral reasoning of girls reflects concerns about responsibility toward others.

174
Q

The mother of a previous client of yours asks for a copy of her daughter’s records. The daughter saw you for two years and died three months ago when she was 38 years old. You should:
Select one:

A.
send the mother a copy of the records as requested.

B.
send the mother only the information you believe will not be harmful.

C.
allow the mother to inspect the records in your office.

D.
not release any information to the mother without appropriate authorization.

A

Laws related to this situation vary from jurisdiction to jurisdiction but, in general, confidentiality does not end when a current or former client dies.
a. Incorrect See explanation for response d.

b. Incorrect See explanation for response d.
c. Incorrect See explanation for response d.
d. CORRECT This is the best course of action of those listed in the answers. In general, a psychologist should not release information after a client’s death without proper authorization from the client’s legal representative. For a discussion of this issue, see R. I. Simon, Clinical psychiatry and the law, American Psychiatric Publishing, Washington, DC, 2003.

The correct answer is: not release any information to the mother without appropriate authorization.

175
Q

Leslie L., age 25, tells her therapist that she recently turned down a job promotion because it would require her to interact more with co-workers and she’s afraid of their criticism. She says she has one friend from high school that she spends some time with but hasn’t made any new friends because she’s always been shy and unpopular and people don’t seem to like her. Leslie says she spends a lot of time fantasizing about “Mr. Right.” Leslie’s symptoms are most suggestive of:
Select one:

A.
Schizoid Personality Disorder.

B.
Schizotypal Personality Disorder.

C.
Borderline Personality Disorder.

D.
Avoidant Personality Disorder.

A

Answer D is correct: Avoidant Personality Disorder is characterized by a pervasive pattern of social inhibition, feelings of inadequacy, and hypersensitivity to negative evaluation. People with this disorder avoid interpersonal contact because they fear criticism, but they desire intimacy and often fantasize about idealized relationships.

Answer A: People with Schizoid Personality Disorder are uninterested in interpersonal relationships and wouldn’t, for example, fantasize about “Mr. Right.”

Answer B: This woman is not exhibiting the kinds of oddities in behavior that are characteristic of Schizotypal Personality Disorder.

Answer C: A person with Borderline Personality Disorder exhibits instability in interpersonal relationships.

The correct answer is: Avoidant Personality Disorder.

176
Q

Kopta and his colleagues (1994) reviewed the recovery rates for 854 psychotherapy clients and found that, for acute symptoms, nearly 75% of the patients showed an alleviation of symptoms (“recovery”) after the 28th therapy session. Their findings are:
Select one:

A.
inconsistent with earlier research by Howard et al. (1986) which found that only 15% of therapy clients showed substantial improvement after the 26th session.

B.
inconsistent with earlier research by Howard et al. (1986) which found that only 48% of therapy clients showed substantial improvement in symptoms after the 26th session.

C.
inconsistent with earlier research by Howard et al. (1986) which found that nearly 96% of therapy clients showed substantial improvement in symptoms after the 26th session.

D.
consistent with earlier research by Howard et al. (1986) which found that 75% of therapy clients showed substantial improvement in symptoms after the 26th session.

A

This question requires you to be familiar with Howard et al.’s research on the “dose-dependent effect” of psychotherapy, which is summarized in the Clinical Psychology chapter of the written study materails.
a. Incorrect See explanation for response d.

b. Incorrect See explanation for response d.
c. Incorrect See explanation for response d.
d. CORRECT K. I. Howard, S. M. Kopta, M. S. Krause, and D. E. Orlinsky found that after the 26th therapy session, 75% of therapy clients included in their study showed a marked improvement in symptoms (The dose-effect relationship in psychotherapy, American Psychologist, 41, 159-164, 1986).

The correct answer is: consistent with earlier research by Howard et al. (1986) which found that 75% of therapy clients showed substantial improvement in symptoms after the 26th session.

177
Q

Research investigating the relationship between perceived exposure to discrimination and mental health has generally found:
Select one:

A.
perceived discrimination is related to depression, anxiety, and other mental disorders among African Americans but not among Whites.

B.
perceived discrimination is related to depression, anxiety, and other mental disorders among both African Americans and Whites.

C.
perceived discrimination is related to physical health among African Americans and Whites but not to mental health for members of either group.

D.
perceived discrimination is not related to either physical health or mental health among African Americans or Whites.

A

The research has found that exposure to discrimination - like exposure to many other stressors - is related to mental health outcomes.
a. Incorrect This issue was addressed by R. C. Kessler, K. D. Mickelson, and D. R. Williams who found that African Americans reported higher levels of perceived exposure to discrimination - e.g., 50% of African Americans and 31% of Whites reported exposure to “major discrimination.” However, the relationship between discrimination and mental health did not vary in a consistent or substantial way across different groups [The prevalence, distribution, and perceived discrimination in the United States, Journal of Health and Social Behavior, 40(3), 208-230, 1999].

b. CORRECT Kessler et al. found a relationship between perceived exposure to discrimination and a diagnosis of Major Depression or Generalized Anxiety Disorder in the previous year for both African Americans and Whites.
c. Incorrect See explanation for response b.
d. Incorrect See explanation for response b.

The correct answer is: perceived discrimination is related to depression, anxiety, and other mental disorders among both African Americans and Whites.

178
Q

Zimbardo’s (1972) prison study confirmed that:
Select one:

A.
uncharacteristic behaviors can be situationally elicited.

B.
behavior is often mediated by affective responses.

C.
people judge their own behavior much like they judge the behavior of others.

D.
the need to conform can precipitate uncharacteristic behaviors.

A

Zimbardo found that a person’s behavior was affected by his assigned role and the demands of the situation.
a. CORRECT Of the responses given, this one best describes Zimbardo’s conclusion. In his prison study, students assigned the role of guard quickly began acting authoritarian, while those assigned the role of prisoner took on passive, helpless behaviors.

b. Incorrect See explanation for response a.
c. Incorrect See explanation for response a.
d. Incorrect See explanation for response a.

The correct answer is: uncharacteristic behaviors can be situationally elicited.

179
Q

The procedures known as “time-out” and “response cost” share in common which of the following?
Select one:

A.
Both are forms of operant extinction.

B.
Both entail withholding or removing positive stimuli.

C.
Both involve reinforcing alternative behaviors.

D.
Both entail using negative reinforcement.

A

Time out involves removing opportunities for reinforcement for a period of time following a behavior to reduce that behavior. Response cost entails removing positive reinforcement (e.g., a desirable activity) following a behavior to eliminate the behavior.
a. Incorrect See explanation for response b.

b. CORRECT Time out and response cost are both types of negative punishment that involve removing positive reinforcement following a behavior in order to reduce or eliminate that behavior.
c. Incorrect See explanation for response b.
d. Incorrect Negative reinforcement is used to increase (not decrease) a behavior.

The correct answer is: Both entail withholding or removing positive stimuli.

180
Q

Which of the following behavioral techniques is most useful for establishing a response that has a low probability of occurring spontaneously?
Select one:

A.
successive approximation conditioning

B.
differential reinforcement

C.
overcorrection

D.
response cost

A

For the exam, you want to be familiar with the four techniques listed in the answers to this question. These techniques are described in the Learning Theory chapter of the written study materials.
a. CORRECT Successive approximation conditioning is another name for shaping, which is used to “shape” responses that are unlikely to occur spontaneously.

b. Incorrect See explanation for response a.
c. Incorrect See explanation for response a.
d. Incorrect See explanation for resopnse a.

The correct answer is: successive approximation conditioning

181
Q

A primary assumption underlying the use of survey feedback as a method of organizational development is that:
Select one:

A.
in order for change to occur, employees must understand the organizations current strengths and weaknesses.

B.
in order for change to occur, barriers to openness and honesty between employees and management must be overcome.

C.
the first step in producing change in an organization is to improve employee motivation and satisfaction.

D.
the first step in producing change in an organization is to identify the nature of the organizations environment.

A

Survey feedback involves collecting data about an organization, providing the information to employees in small group meetings, and then having employees analyze the identified problems and find solutions to them.
a. CORRECT A primary assumption underlying the technique known as survey feedback is that, for change to occur, employees must be familiar with the organization’s strengths and weaknesses. For this reason, it involves including employees in all steps of the process.

b. Incorrect This sounds more like a premise of sensitivity training.
c. Incorrect This better describes the premise underlying quality of work life programs.
d. Incorrect This best describes an assumption of contingency theory.

The correct answer is: in order for change to occur, employees must understand the organizations current strengths and weaknesses.

182
Q

Research investigating the relationship between ADHD and substance abuse has found that:
Select one:

A.
a diagnosis of ADHD in childhood is not associated with an increased risk for substance abuse in adulthood.

B.
a diagnosis of ADHD in childhood is associated with an increased risk for substance abuse in adulthood.

C.
a diagnosis of ADHD in childhood is associated with an increased risk for substance abuse in adulthood only if the individual was treated with stimulant drugs in childhood.

D.
a diagnosis of ADHD in childhood is associated with an increased risk for substance abuse in adulthood only if the individual had comorbid conduct symptoms.

A

Answer B is correct: ADHD with or without comorbidity is a risk factor for substance abuse in adulthood. See, e.g., J. Biederman et al., Does attention-deficit hyperactivity disorder impact the developmental course of drug and alcohol abuse and dependence?, Biological Psychiatry, 44, 269-273, 1998.

Answers A and D: See explanation for answer B.

Answer C: There is evidence that successful treatment with a stimulant drug actually has a protective effect with regard to the development of substance abuse. See, e.g., J. Biederman et al., Pharmacotherapy of attention-deficit/hyperactivity disorder reduces the risk for substance use disorder, Pediatrics, 104, 20, 1999.

The correct answer is: a diagnosis of ADHD in childhood is associated with an increased risk for substance abuse in adulthood.

183
Q

Which of the following is not a rater bias?
Select one:

A.
strictness error

B.
central tendency bias

C.
floor effect

D.
halo effect

A

The accuracy of subjective rating scales is affected by the scale’s susceptibility to rater biases.
a. Incorrect Strictness error (bias) refers to the tendency to rate all ratees using the low end of the rating scale.

b. Incorrect A rater is exhibiting the central tendency bias when he/she rates all ratees using the middle of a rating scale.
c. CORRECT A floor effect is a characteristic of a measuring instrument (not a rater) and refers to the instrument’s inability to distinguish between people who have low levels of whatever is being measured by the instrument.
d. Incorrect A rater is exhibiting a halo effect when his/her evaluation of an employee on one dimension of performance affects the rater’s evaluation of that employee on other unrelated dimensions of performance.

The correct answer is: floor effect

184
Q

: The inability to repeat what one has just heard and to name familiar objects without a loss of comprehension is characteristic of which of the following?
Select one:

A.
Broca’s and Wernicke’s aphasia

B.
Wernicke’s aphasia

C.
conduction aphasia

D.
anomic aphasia

A

The aphasias all involve a disruption in language.
a. Incorrect Broca’s aphasia involves difficulty in producing speech, while Wernicke’s aphasia involves impaired comprehension with fluent aphasia.

b. Incorrect Wernicke’s aphasia is characterized by impairment in comprehension and fluent aphasia.
c. CORRECT Conduction aphasia is narrower in terms of symptoms than Broca’s aphasia and Wernicke’s aphasia. It is the result of damage to the arcuate fasciculus which connects Broca’s and Wernicke’s areas and involves the symptoms listed in the question.
d. Incorrect Anomic aphasia is characterized by impairments in the ability to retrieve and label semantic concepts.

The correct answer is: conduction aphasia

185
Q

Assuming that each score is from a normal distribution of scores, which of the following accurately lists the scores in order of magnitude from lowest to highest?
Select one:

A.
T = 40; z = 0; PR = 82; WAIS-IV IQ = 126

B.
PR = 16; T = 30; z = 1.5; WAIS-IV IQ = 116

C.
z = -2; T = 25; WAIS-IV IQ = 115; PR = 75

D.
PR = 50; T = 50; WAIS-IV IQ = 132; z = 1.5

A

Answer A is correct: For the exam, you want to be familiar with the relationship between norm-referenced scores in a normal distribution. A figure illustrating their relationship is provided in the Test Construction chapter of the written study materials. A T score of 40 is one standard deviation below the mean, a z score of 0 is equal to the mean, a percentile rank (PR) of 82 is slightly below one standard deviation above the mean, and a WAIS-IV score of 126 is slightly below two standard deviations above the mean.

The correct answer is: T = 40; z = 0; PR = 82; WAIS-IV IQ = 126

186
Q

With regard to Schizophrenia, there is some evidence that the higher rate found among African American individuals may be due to:
Select one:

A.
genetic factors.

B.
misdiagnosis.

C.
lower recovery rates.

D.
a higher risk for complications during birth.

A

Answer B is correct: Although the research on the relation between race/ethnicity and Schizophrenia is limited, there is some evidence that African American individuals have a slightly higher rate of Schizophrenia than other groups. And the results of the existing studies suggest that African Americans with other disorders (especially Bipolar Disorder, depression, and alcohol-related organic disorders) are often misdiagnosed as having Schizophrenia because their symptoms include auditory and visual hallucinations. See, e.g., NIMH Epidemiologic Catchment Area Study, reported in American Psychiatric Association, Ethnic Minority Elderly: A Task Force Report of the American Psychiatric Association, Washington, DC, APA, 1994.

Answers A, C, and D: See explanation for answer B.

The correct answer is: misdiagnosis.

187
Q

Dr. Eliana Escobar is appointed by the court to evaluate a defendant in a criminal case to determine his state of mind at the time he committed the crime. In this situation, Dr. Escobar:
Select one:

A.
must obtain informed consent from the defendant prior to the evaluation.

B.
must obtain informed consent from the defendant or the defendant’s attorney prior to the evaluation.

C.
does not have to obtain an informed consent but must notify the defendant of the purpose of the evaluation.

D.
does not have to provide information to the defendant unless requested to do so by the court or the defendant’s attorney.

A

This issue is addressed in the APA’s Specialty Guidelines for Forensic Psychology.
a. Incorrect See explanation for response c.

b. Incorrect See explanation for response c.
c. CORRECT Although a psychologist is not required to obtain an informed consent from a defendant when an evaluation is court-ordered, he/she should inform the defendant of the purpose of the evaluation and of the limits on confidentiality before conducting it.
d. Incorrect See explanation for response c.

The correct answer is: does not have to obtain an informed consent but must notify the defendant of the purpose of the evaluation.

188
Q

A test developer would use Lord’s chi-square in order to:
Select one:

A.
compare the standard error of measurement for two different tests or test items.

B.
predict the number of people who are likely to answer a test item correctly.

C.
determine whether factors in a factor analysis are orthogonal or oblique.

D.
evaluate the differential item functioning of an item included in a test.

A

The chi-square test serves numerous functions.
a. Incorrect See explanation for response d.

b. Incorrect See explanation for response d.
c. Incorrect See explanation for response d.
d. CORRECT In the context of item response theory, differential item functioning (DIF) is another name for item bias and is occurring when one group responds differently to an item than another group even though both groups have similar levels of the latent trait (attribute) measured by the test. Several statistical techniques are used to evaluate DIF. Lord’s chi-square is one of these techniques.

The correct answer is: evaluate the differential item functioning of an item included in a test.

189
Q

Dr. Rob Roberts, a licensed psychologist, receives a call from a former client several months after the client terminated therapy. The former client tells Dr. Roberts that he has been seeing a psychiatrist for the past two months and that the psychiatrist has prescribed medication as a treatment for his depression. The man says he doesn’t think the medication is helping him at all and that he doesn’t really like the psychiatrist. He says that he wants to stop taking the medication and start therapy again with Dr. Roberts. The best course of action for Dr. Roberts would be to:
Select one:

A.
make an appointment with the man to discuss the matter further.

B.
advise the man that it would be best if he discusses the matter with the psychiatrist before making an appointment with Dr. Roberts.

C.
make an appointment with the man and, if the medication seems to be having no effect, call the psychiatrist to discuss the matter with her.

D.
tell the man that he (Dr. Roberts) will call the psychiatrist to discuss the matter with her.

A

In most situations, psychologists currently cannot prescribe medication or give advice about medication. Therefore, the correct answer to this question must be the answer that involves advising the client to speak with the psychiatrist who prescribed the medication.
a. Incorrect Since Dr. Roberts is not able to make recommendations about medications, this is not the best course of action.

b. CORRECT This is the only response that involves advising the client to check with the psychiatrist about the continued use of the medication and, therefore, is the best answer.
c. Incorrect Since prescribing medications is not within Dr. Roberts’ area of expertise, this is not an appropriate course of action.
d. Incorrect Again, since Dr. Roberts is not able to give advice about medications, this is not the best choice.

The correct answer is: advise the man that it would be best if he discusses the matter with the psychiatrist before making an appointment with Dr. Roberts.

190
Q

Which of the following best describes the effects of overlearning?
Select one:

A.
good long-term recall and resistance to extinction

B.
good short-term recall but no effect on long-term recall or extinction

C.
slight detrimental effects on recall

D.
no significant effects on short-or long-term recall

A

Overlearning refers to continued practice beyond the point of mastery.

a. CORRECT Overlearning, or practicing a behavior beyond the time required to learn it, is associated with improvements in both short-and long-term recall and resistance to extinction.
b. Incorrect Overlearning has shown to be effective for short-term and long-term recall. Overlearning is also associated with resistance to extinction.
c. Incorrect There are no detrimental effects on recall with overlearning.
d. Incorrect Overlearning has positive effects on recall.

The correct answer is: good long-term recall and resistance to extinction

191
Q

If your data analysis involves calculating an “effect size,” you are conducting which of the following?
Select one:

A.
meta-analysis

B.
path analysis

C.
factor analysis

D.
discriminant analysis

A

Only one of the statistical techniques listed in the answers involves calculating an effect size, which is a measure of the magnitude of the relationship between two variables (e.g., a treatment and an outcome).
a. CORRECT Meta-analysis is used to combine the results of multiple studies. For example, a meta-analysis might be conducted to combine the results of several different studies that investigated the effects of cognitive therapy on depression. Use of this technique involves calculating an effect size for each study and then calculating a mean effect size for all of the studies.

b. Incorrect See explanation for response a.
c. Incorrect See explanation for response a.
d. Incorrect See explanation for response a.

The correct answer is: meta-analysis

192
Q

__________ is a type of cost analysis that entails comparing the costs and benefits of several treatment approaches using “quality-adjusted life years” as the measure of treatment benefits.
Select one:

A.
Cost-benefit analysis

B.
Cost-utility analysis

C.
Cost-minimization analysis

D.
Cost-feasibility analysis

A

For the exam, you want to be familiar with the approaches to cost analysis listed in the answers to this question. These are described in the Ethics and Professional Issues chapter in the written study materials.
a. Incorrect See explanation for response b.

b. CORRECT When conducting a cost-utility analysis (CUA), measures of duration and quality of life for each intervention are combined and the result is compared to the monetary costs associated with the intervention. Quality-adjusted life years is a commonly used combined measure.
c. Incorrect See explanation for response b.
d. Incorrect See explanation for response b.

The correct answer is: Cost-utility analysis

193
Q

When working in military settings, a clinical psychologist should keep in mind that:
Select one:

A.
clients should be informed of potential limits to confidentiality at the outset of the delivery of services.

B.
confidential client information should not be revealed to military personnel without the client’s consent except in the “most unusual circumstances.”

C.
client confidentiality should always be treated in the same way that it would be treated in other settings.

D.
legal and ethical requirements related to confidentiality are suspended as required by military rules and policies.

A

To be consistent with ethical requirements regarding client confidentiality, the potential limits on confidentiality should be discussed at the outset of therapy in all settings.
a. CORRECT Client confidentiality is more limited in military settings than it is in other settings, and clients should be informed of these limits during the informed consent process.

b. Incorrect This answer does not accurately describe a psychologist’s ethical obligations regarding client confidentiality in military settings.
c. Incorrect Client confidentiality is not treated exactly the same way in military settings as it is in other settings.
d. Incorrect All requirements related to confidentiality are not suspended in military settings. For example, psychologists are still ethically obligated to discuss the limits on confidentiality with clients.

The correct answer is: clients should be informed of potential limits to confidentiality at the outset of the delivery of services.

194
Q

earch participants assigned the role of teacher would deliver electric shock to another person when the experimenter:
Select one:

A.
let them know they were not really harming the learner.

B.
took responsibility for the teacher’s actions.

C.
modeled the behavior by delivering high levels of shock to the learner.

D.
rewarded them for doing so.

A

Participants in Milgram’s (1963) obedience to authority studies were assigned the role of teacher and asked to deliver electric shock to a learner (actually a confederate) when the learner made a mistake while recalling a list of words.
a. Incorrect No participants were told that they were not actually hurting the confederate.

b. CORRECT Of the answers given, this one best describes a conclusion that can be drawn from Milgram’s research - i.e., participants believed the experimenter was responsible for the events that occurred during the study.
c. Incorrect The experimenter did not shock the learner during the course of the study.
d. Incorrect Participants were not rewarded by the experimenter for shocking the learner.

The correct answer is: took responsibility for the teacher’s actions.

195
Q

The primary goal of quality assurance is best described as:
Select one:

A.
reducing program costs and beneficiary expenditures.

B.
improving the health status and satisfaction of patients.

C.
policing the profession by detecting and dealing appropriately with incompetent psychologists.

D.
guaranteeing that the needs of patients belonging to a particular group or population are being met.

A

As its name implies, the focus of quality assurance is on the quality of the services provided.
a. Incorrect This better describes the goal of “utilization review.”

b. CORRECT Quality assurance is directed toward ensuring that programs and services are effective, efficient, and available. This goal is achieved by comparing services to predefined standards.
c. Incorrect This could be one of the outcomes of quality assurance, but it is certainly not its primary goal.
d. Incorrect This answer isn’t as good as response b because quality assurance can’t guarantee that the needs of patients are being met.

The correct answer is: improving the health status and satisfaction of patients.

196
Q

To calculate a predictor’s sensitivity, you would:
Select one:

A.
divide the number of true positives by the number of true and false positives.

B.
divide the number of true negatives by the number of true and false negatives.

C.
divide the number of true positives by the number of true positives plus false negatives.

D.
divide the number of true negatives by the number of true negatives plus false positives.

A

Sensitivity refers to the percent of cases in the validation sample that have the attribute being assessed and were accurately classified as having the attribute by the predictor.
a. Incorrect This is the formula for the positive predictive value.

b. Incorrect This is the formula for the negative predictive value.
c. CORRECT This is the formula for calculating sensitivity.
d. Incorrect This describes the formula for calculating specificity.

The correct answer is: divide the number of true positives by the number of true positives plus false negatives.

197
Q

A current client has filed a complaint against you with the Ethics Committee and the Committee has contacted you requesting that you provide it with test results and other client information that it needs to resolve the client’s complaint. You have attempted to contact the client to obtain his release but he has not responded to your attempts and has stopped coming to therapy. You believe that the client’s complaint is frivolous and unfounded. You should:
Select one:

A.
provide the Committee with the information it has requested.

B.
provide the Committee only with information you believe is relevant to the complaint.

C.
contact the Committee and inform it about your inability to obtain a release from the client.

D.
contact the Committee and inform it that you believe the client’s complaint is improper.

A

This issue is addressed in Standard 1.06 of the APA’s Ethics Code and by the Preamble (Responsibiity of the Individual Psychologist) and requirements related to confidentiality in the Canadian Code of Ethics for Psychologists.
a. Incorrect See explanation for response c.

b. Incorrect See explanation for response c.
c. CORRECT This response is most consistent with ethical requirements. For example, Standard 1.06 states: “Psychologists cooperate in ethics investigations … [and] in doing so, they address any confidentiality issues.”
d. Incorrect See explanation for response c.

The correct answer is: contact the Committee and inform it about your inability to obtain a release from the client.

198
Q

As defined by Piaget, egocentrism during the preoperational stage of cognitive development is a manifestation of:
Select one:

A.
primary circular reactions.

B.
transduction.

C.
horizontal decalage.

D.
centration.

A

Piaget distinguished between four stages of cognitive development. The preoperational stage extends from ages 3 to 6 years.
a. Incorrect Primary circular reactions are characteristic of babies between the ages of 1 and 4 months and involve the repetition of pleasurable actions.

b. Incorrect Transductive thinking is characteristic of the preoperational stage. However, as defined by Piaget, it refers to reasoning characterized by a tendency to move from one particular case to another particular case without taking the general into account (e.g., I had bad thoughts about my mom; she got sick; therefore, my thoughts made her sick).
c. Incorrect Horizontal decalage refers to the inability to transfer one type of conservation to another (e.g., the conservation of substance to weight or volume).
d. CORRECT Knowing that Piaget described centration as a primary limitation of the preoperational stage would have helped you identify this as the correct answer. Piaget believed that egocentrism is a form of centration - i.e., young children cannot take into account their own point of view and the point of view of another person at the same time.

The correct answer is: centration.

199
Q

A person who is low in self-monitoring (Snyder, 1987) will rely on which of the following when deciding how to act in a particular social situation?
Select one:

A.
his/her life scripts.

B.
his/her own feelings, attitudes, and beliefs.

C.
the emotional reactions and behaviors of the people he/she is interacting with.

D.
internal working models that were acquired during infancy and early childhood.

A

As defined by Snyder (1987), self-monitoring refers to the degree to which people are concerned about the impressions they convey to others in social situations and are able to control those impressions.
a. Incorrect See explanation for response b.

b. CORRECT A person who is low in self-monitoring relies on his/her own feelings, attitudes, and beliefs when determining how to act in social situations and, as a result, acts similarly in different social situations.
c. Incorrect People who are high in self-monitoring are concerned about the impressions that others have of them and, as a result, adapt their behaviors to the emotional reactions and behaviors of the people in each social situation.
d. Incorrect See explanation for response b.

The correct answer is: his/her own feelings, attitudes, and beliefs.

200
Q

You’ve been seeing a client for three months. During that time, he has complained several times about the course of therapy and, each time, you have discussed this matter with him and modified treatment goals. During the last two sessions, he has again mentioned that he is not happy with therapy. As an ethical psychologist, your best course of action would be to:
Select one:

A.
realize that his behavior is a manifestation of resistance and ignore it.

B.
make a clinical decision as to whether or not the client is benefitting from therapy and, if you determine that he is, encourage him to continue in therapy.

C.
discuss his dissatisfaction with him and the possibility of a referral to another therapist.

D.
tell him that, as an ethical therapist, you must terminate treatment since he does not feel he is benefitting.

A

Since this is an ethics question, you can eliminate answers a and b because they deal more with a clinical issue.
a. Incorrect See explanation above.

b. Incorrect See explanation above.
c. CORRECT This is a better answer than response d because it more clearly takes the welfare of the client into consideration. It is also the more conservative answer, and a good strategy for ethics questions is to pick the more conservative course of action.
d. Incorrect This isn’t as good as response c because it comes too close to being client abandonment.

The correct answer is: discuss his dissatisfaction with him and the possibility of a referral to another therapist.

201
Q

Studies comparing the communication styles of men and women suggest that:
Select one:

A.
in comparison to men, women interrupt others more often.

B.
in comparison to women, men interrupt others more often.

C.
in comparison to men, women interrupt women (but not men) more often.

D.
in comparison to women, men interrupt men (but not women) more often.

A

This is another one of those questions that is difficult to answer because the research is far from consistent.
a. Incorrect See explanation for response b.

b. CORRECT Of the responses given, this is the best one. Males are more likely to do the interrupting, while females are more likely to be the “victims” of interruptions.
c. Incorrect See explanation for response b.
d. Incorrect See explanation for response b.

The correct answer is: in comparison to women, men interrupt others more often.

202
Q

Following a stroke involving the middle cerebral artery in the right (non-dominant) hemisphere, a person is most likely to exhibit:
Select one:

A.
paralysis on the left side of his body and vision loss in his left visual field.

B.
paralysis on the left side of his body and global aphasia.

C.
paralysis on the right side of his body and visual agnosia.

D.
paralysis on the right side of his body and visual loss in his right visual field.

A

Knowing that the right hemisphere controls the left side of the body and the left visual field would have helped you choose the correct answer to this question.

a. CORRECT Damage to certain areas of the right hemisphere could cause paralysis on the left side of the body as well as vision loss involving the left visual field.
b. Incorrect In most people, language is mediated by the left hemisphere so that damage to the right hemisphere would not cause aphasia (language disturbances).
c. Incorrect See explanation above.
d. Incorrect See explanation above.

The correct answer is: paralysis on the left side of his body and vision loss in his left visual field.

203
Q

In a research study designed to evaluate the accuracy of a selection test for police officers, the test is found to have high validity coefficients for Whites, African Americans, and Asian Americans, but a low validity coefficient for American Indians. In this situation, race/ethnicity is a:
Select one:

A.
suppressor variable.

B.
moderator variable.

C.
distractor variable.

D.
mediator variable.

A

In this situation, the predictor appears to have differential validity–i.e., different validity coefficients for different groups.
a. Incorrect See explanation for response b.

b. CORRECT The characteristic that distinguishes the groups is referred to as a moderator variable. The moderator variable (in this case, ethnicity) “moderates” the relationship between the predictor and criterion.
c. Incorrect See explanation for response b.
d. Incorrect See explanation for response b.

The correct answer is: moderator variable.

204
Q

You are currently treating a man who has received a diagnosis of Alcohol Use Disorder. With his permission, you invite his wife to participate in couple’s therapy. Inviting the wife to participate in therapy is:
Select one:

A.
unacceptable because, in this situation, the husband is “vulnerable to undue influence.”

B.
unacceptable because this situation represents a “conflict of interest.”

C.
acceptable as long as the husband is not receiving treatment from another professional.

D.
acceptable as long as you believe her participation will benefit the husband’s progress in treatment.

A

This issue is addressed in Standard 5.06 of the APA’s Ethics Code and Principles I.27 and III.31 of the Canadian Code of Ethics for Psychologists. a. Incorrect See explanation for response d. b. Incorrect See explanation for response d. c. Incorrect See explanation for response d. d. CORRECT Although it is generally unacceptable to solicit business from individuals who are “vulnerable to undue influence” (answer a), there are exceptions. For example, Standard 5.06 states that inviting family members to participate in collateral treatment that will benefit a current client is acceptable.

The correct answer is: acceptable as long as you believe her participation will benefit the husband’s progress in treatment.

205
Q

As described by Piaget, using objects in novel ways and deliberately varying an action to provoke new results is characteristic of which sensorimotor substage?
Select one:

A.
primary circular reactions

B.
secondary circular reactions

C.
coordinated secondary reactions

D.
tertiary circular reactions

A

As described in the Lifespan Development chapter of the written study materials, Piaget distinguished between six sensorimotor substages.
a. Incorrect Primary circular reactions are simple motor habits that involve the infant’s body (e.g., thumbsucking).

b. Incorrect Secondary circular reactions include simple repetitive actions that help the infant gain volunatary control over his/her behavior.
c. Incorrect Coordinated secondary reactions include intentional, goal-directed behaviors that combine secondary circular reactions into new, more complex action sequences.
d. CORRECT Tertiary circular reactions involve exploring the properties of objects by acting on them in new ways – e.g., twisting and turning a shaped object so that it fits through a hole.

The correct answer is: tertiary circular reactions

206
Q

A 67-year-old accountant exhibits impairments in memory and other cognitive functions along with a depressed mood. The presence of which of the following suggests that his symptoms are due to “pseudodementia” (depression) rather than mild Neurocognitive Disorder?
Select one:

A.
The onset of his symptoms was gradual.

B.
The man tends to exaggerate his cognitive problems.

C.
The man’s symptoms worsen over the course of the day.

D.
The man exhibits greater impairment in declarative memory than procedural memory.

A

Answer B is correct: Depression is often difficult to differentiate from mild Neurocognitive Disorder when the depression includes prominent cognitive symptoms. However, there are some distinguishing characteristics. For example, people with depression are likely to exaggerate their cognitive problems, while those with mild Neurocognitive Disorder are likely to minimize or deny them.

Answers A, C, and D: A gradual onset of symptoms and a worsening of symptoms in the evening are more characteristic of Neurocognitive Disorder. Also, people with depression tend to exhibit greater impairment in procedural memory than declarative memory.

The correct answer is: The man tends to exaggerate his cognitive problems.

207
Q

A “speed” test can be viewed as the opposite of a _______ test.
Select one:

A.
mastery

B.
performance

C.
power

D.
context

A
A speed (or speeded) test contains a large number of questions that are so easy that most examinees would be able to answer them correctly if given sufficient time. However, a strict time limit is imposed so that an examinee's test score reflects his/her speed of responding.
a. Incorrect See explanation for response c.

a. Incorrect See explanation for response c.
c. CORRECT Power tests can be viewed as the opposite of speed tests. Power tests contain items that are relatively difficult and that could not all be answered correctly by most or all examinees regardless of how much time they are given to complete the test.
d. Incorrect See explanation for response c.

The correct answer is: power

208
Q

Research on the “synchrony effect” has established a relationship between circadian arousal, cognitive functioning, and age. These studies have found, for example, that the performance of older adults on cognitive tasks that require the ability to focus on goal-relevant information only and to inhibit prepotent responses is best in the:
Select one:

A.
morning.

B.
early afternoon.

C.
early evening.

D.
late evening.

A

Studies have found that the optimal time for the successful completion of certain types of cognitive tasks is related to circadian arousal, which varies with age: Older adults show peak arousal and task performance levels in the morning, while young adults show higher levels of both in the evening.
a. CORRECT Older adults do better on certain cognitive tasks (especially those that depend on inhibitory attentional processes) in the morning than during any other time of the day.

b. Incorrect See explanation above.
c. Incorrect See explanation above.
d. Incorrect See explanation above.

The correct answer is: morning.

209
Q

Which of the following accurately describes the utilization rates for mental health services among Whites and African Americans?
Select one:

A.
Compared to Whites, African Americans have higher utilization rates for both inpatient and outpatient mental health services.

B.
Compared to Whites, African Americans have lower utilization rates for both inpatient and outpatient mental health services.

C.
Compared to Whites, African Americans have lower rates for outpatient mental health services but higher rates for public inpatient mental health services.

D.
Compared to Whites, African Americans have higher rates for outpatient mental health services but lower rates for public inpatient mental health services.

A

As noted in the Clinical Psychology chapter of the written study materials, research by Snowden et al. and others has consistently identified disparities in the utilization rates for some ethnic minority groups when compared to the rates for Whites.
a. Incorrect See explanation for response c.

b. Incorrect See explanation for response c.
c. CORRECT Snowden et al.’s data indicate that, in comparison to Whites, African Americans are overrepresented in public inpatient mental health services but are underrepresented in outpatient mental health services. See, e.g., L. R. Snowden, African American service use for mental health problems, Journal of Community Psychology, 27(3), 303-313, 1999.
d. Incorrect See explanation for response c.

The correct answer is: Compared to Whites, African Americans have lower rates for outpatient mental health services but higher rates for public inpatient mental health services.

210
Q

The view that depression is the result of exposure to an aversive event coupled with the tendency to attribute negative events to personal, global, and stable factors is consistent with:
Select one:

A.
the reformulated learned helplessness model.

B.
the fundamental attribution bias.

C.
the original aversive events model.

D.
Rehm’s self-control model.

A

Answer A is correct: In the late 1970s, the learned helplessness model was expanded to include the role of attributions in the development of depression. (Note that, as described in the Abnormal Psychology chapter of the written study materials, the most recent version of the model places less emphasis on attributions and, instead, focuses on the role of hopelessness.)

Answer B: The fundamental attribution bias refers to the tendency to attribute the behavior of others to dispositional factors.

Answer C: This is a “made-up” term.

Answer D: Rehm’s self-control model views depression as being the result of problems related to self-monitoring, self-evaluation, and self-reinforcement.
The correct answer is: the reformulated learned helplessness model.

211
Q

When taking a practice test in preparation for the licensing exam, your performance on the test is least likely to be adversely affected by:
Select one:

A.
a loudly ticking clock.

B.
the stereo that you’re too lazy to turn off.

C.
neighbors arguing in the apartment next door.

D.
a thunderstorm.

A

Responses to different kinds of noise vary from individual to individual, but the research has revealed some consistencies.
a. Incorrect See explanation for response b.

b. CORRECT Noise that can be controlled (stopped) has less adverse effects on task performance even when it is not actually terminated. Apparently it is the sense of control over the noise that is the important factor. Intermittent noises and noises unrelated to the task are likely to have a negative effect.
c. Incorrect See explanation for response b.
d. Incorrect See explanation for response b.

The correct answer is: the stereo that you’re too lazy to turn off.

212
Q

Based on his research with rats in mazes, Edward Tolman (1938) concluded that the rats had developed cognitive maps as the result of:
Select one:

A.
latent learning.

B.
insight learning.

C.
vicarious learning.

D.
associative learning.

A

Tolman found that rats who had been allowed to explore the maze without reinforcement were able to reach the end of the maze faster when reinforcement was provided than did rats who had not been allowed to explore the maze.
a. CORRECT Tolman concluded that rats that had been allowed to explore the maze had formed cognitive maps (mental representations) of the maze and confirmed that latent learning (learning without reinforcement) can occur.

b. Incorrect See explanation above.
c. Incorrect See explanation above.
d. Incorrect See explanation above.

The correct answer is: latent learning.

213
Q

Deficits in behavioral inhibition associated with ADHD have been linked to abnormalities in which of the following brain structures?
Select one:

A.
hypothalamus

B.
reticular activating system

C.
cingulate gyrus

D.
caudate nucleus

A

Neuroimaging has made it possible to localize brain structures associated with specific symptoms of ADHD and other disorders.
a. Incorrect See explanation for response d.

b. Incorrect See explanation for response d.
c. Incorrect See explanation for response d.
d. CORRECT ADHD - and, in particular, the lack of behavioral inhibition associated with ADHD - has been linked to an abnormally small caudate nucleus, globus pallidus, and prefrontal cortex.

The correct answer is: caudate nucleus

214
Q

A psychologist is hired to develop a community program for seniors that will focus on maintaining social contacts and reducing the risk of social isolation. This program is an example of which of the following?
Select one:

A.
milestone prevention

B.
primary prevention

C.
secondary prevention

D.
tertiary prevention

A

In this situation, a program is being developed for a particular population rather than specific individuals from that population.
a. Incorrect Bloom (1968) distinguishes between community-wide, milestone, and high-risk preventions. Milestone programs are aimed at people experiencing a transition (e.g., retirement).

b. CORRECT Since this program is aimed at all seniors in the community, it is a primary prevention.
c. Incorrect A secondary prevention would be aimed at specific individuals who have been identified as being at high-risk. As originally defined, secondary prevention always involves some kind of screening process to identify individuals.
d. Incorrect A tertiary program is for people who have already suffered mental health problems. Its purpose is to reduce the risk for relapse or a continuation or worsening of the problem.

The correct answer is: primary prevention

215
Q

A 6th grader goes through the motion of reading his history text without realizing that he has failed to understand the material he has just read. This:
Select one:

A.
is indicative of low intelligence.

B.
is indicative of a learning disability.

C.
illustrates a lack of tacit knowledge.

D.
illustrates poor metacognitive skills.

A

In this situation, the student is apparently unaware of his own cognitive processes.
a. Incorrect This is not necessarily a sign of low intelligence.

b. Incorrect This is not necessarily a sign of a learning disability.
c. Incorrect Tacit knowledge refers to strategies for success that aren’t usually formally taught.
d. CORRECT The inability to monitor one’s own cognitive processes suggests poor metacognitive (“knowing about knowing”) skills.

The correct answer is: illustrates poor metacognitive skills.

216
Q

The representative of a client’s insurance company asks you to fax diagnosis and treatment information about the client to her and the client has already signed a release of information. As an ethical psychologist, you should:
Select one:

A.
make sure that all information that might identify the client is coded or removed.

B.
make sure the record is addressed to the representative and is clearly marked “confidential.”

C.
send a hard copy (written) version of the file to the representative as a follow-up to your fax.

D.
refuse to fax the information but send a hard copy instead.

A

This issue is addressed in Standard 6.02(b) of the APA’s Ethics Code and Principle I.42 of the Canadian Code of Ethics.
a. CORRECT Standard 6.02(b) states: “If confidential information concerning recipients of psychological services is entered into databases or systems of records available to persons whose access has not been consented to by the recipient, psychologists use coding or other techniques to avoid the inclusion of personal identifiers.” Since it is not possible to ensure that anyone other than the representative will not have access to the faxed material, it would be important to delete or code all identifying information.

b. Incorrect See explanation for response a.
c. Incorrect See explanation for response a.
d. Incorrect See explanation for response a.

The correct answer is: make sure that all information that might identify the client is coded or removed.

217
Q

When interpreting an examinee’s responses to the Rorschach test, form quality provides information on which of the following?
Select one:

A.
coping resources

B.
interpersonal style

C.
cognitive style

D.
reality testing

A

Form quality is one of several factors considered when scoring the Rorschach.
a. Incorrect Coping resources are indicated by responses related to human and animal movement.

b. Incorrect Interpersonal style is assessed by looking at the proportion of human versus animal responses.
c. Incorrect Cognitive style is evaluated by considering under- versus overincorporation.
d. CORRECT Form quality refers to the similarity between the examinee’s perception of the inkblot and its actual shape and provides information on reality testing.

The correct answer is: reality testing

218
Q

Performance on which of the following is least likely to be adversely affected by increasing age in adulthood?
Select one:

A.
a measure of implicit memory

B.
a dichotic listening task

C.
a measure of processing speed

D.
a test of visual acuity

A

Increasing age is associated with decrements in a number of cognitive and physical abilities.
a. CORRECT Although explicit memory shows age-related declines, implicit memory is relatively unaffected by increasing age.

b. Incorrect Dichotic listening assesses divided attention which does show age-related declines.
c. Incorrect Slowing in the speed of processing and responding is one of the most consistent findings of research investigating the impact of increasing age on cognitive abilities.
d. Incorrect Visual acuity begins to decline somewhat in the 40s and then shows a rapid decline between 60 and 80.

The correct answer is: a measure of implicit memory

219
Q

Dr. Alicia Alvarez routinely administers the MMPI-2 as part of the initial evaluation of her adult therapy clients. She also uses the test data in her ongoing research study. This practice is:
Select one:

A.
unethical since this represents a conflict of interest.

B.
unethical unless she has obtained each client’s consent to participate in the research study.

C.
ethical as long as each client’s confidentiality is maintained.

D.
ethical as long as the study has been approved by the institutional review board.

A

The correct answer is B. This issue is addressed in Standard 9.03(a) of the APA’s Ethics Code. It would be necessary to obtain informed consent from the clients before conducting the assessment, and the consent must include “an explanation of the nature and purpose of the assessment.” In this situation, the clients apparently believe that they are being tested for diagnostic and treatment purposes and to use the test data for other reasons without their consent would be unethical.

The correct answer is: unethical unless she has obtained each client’s consent to participate in the research study.

220
Q

When a research study will cause participants pain, deception:
Select one:

A.
is prohibited if the pain is physical but not if it is psychological in nature.

B.
is prohibited whether the pain is physical or psychological in nature.

C.
is acceptable only if no alternative methods for obtaining the data are available.

D.
is acceptable only if it is justified by the study’s “prospective value.”

A

Deception in research is acceptable only when certain conditions are met.
a. Incorrect See explanation for response b.

b. CORRECT This response is most consistent with Standard 8.07(b), which states, “Psychologists do not deceive prospective participants about research that is reasonably expected to cause physical pain or severe emotional distress.”
c. Incorrect See explanation for response b.
d. Incorrect See explanation for response b.

The correct answer is: is prohibited whether the pain is physical or psychological in nature.

221
Q

From smallest to largest in magnitude, the measures of central tendency in negatively skewed disribution are:
Select one:

A.
mode, median, and mean.

B.
median, mode, and mean.

C.
mode, mean, and median.

D.
mean, median, and mode.

A

Knowing that the median is always the middle score would have allowed to narrow the choices to answers a and d.
a. Incorrect This describes the relationship of the measures of central tendency in a positively skewed distribution.

b. Incorrect See explanation for response d.
c. Incorrect See explanation for response d.
d. CORRECT Remember that the “tail tells the tale,” which means that, in a negatively skewed distribution, most scores are located in the positive (high) end of the distribution and the tail is created by the presence of a few scores in the negative (low) end of the distribution. Since the mean is most affected by the presence of the outliers in the negative end, it will have the lowest value of the three measures of central tendency.

The correct answer is: mean, median, and mode.

222
Q

Dissociative Amnesia most often involves which of the following types of memory loss?
Select one:

A.
localized and selective

B.
generalized and continuous

C.
systematized and selective

D.
continuous and global

A

Answer A is correct: According to the DSM-5, localized and selective amnesia for one or more events are the most common types of memory loss associated with Dissociative Amnesia.

Answers B, C, and D: See explanation for answer A.

The correct answer is: localized and selective

223
Q

With regard to the DSM, the term “polythetic” refers to:
Select one:

A.
the fact that a diagnosis can be assigned to an individual when he/she exhibits only a subset of the characteristic symptoms for that diagnosis.

B.
the fact that each diagnosis is characterized by both core and associated symptoms.

C.
the requirement that culture and other individual factors be considered before assigning a specific diagnosis.

D.
the fact that some individuals have symptoms that fit the diagnostic criteria for two or more disorders.

A

Answer A is correct: The DSM uses a categorical diagnostic system and allows for heterogeneity within each diagnostic category through its use of a polythetic criteria set that requires an individual to meet only a subset of characteristic symptoms.

Answers B, C, and D: See explanation for answer A.

The correct answer is: the fact that a diagnosis can be assigned to an individual when he/she exhibits only a subset of the characteristic symptoms for that diagnosis.

224
Q

When the Kaufman Assessment Battery for Children, Second Edition (KABC-II) is administered to a child with a language or hearing impairment, his/her scores are interpreted in terms of which model of intelligence?
Select one:

A.
Spearman’s

B.
Luria’s

C.
Gardner’s

D.
Sternberg’s

A

Scores on the KABC-II may be interpreted in terms of the Cattell-Horn-Carroll (CHC) model of cognitive abilities or Luria’s neuropsychological processing model.
a. Incorrect See explanation for response b.

b. CORRECT Luria’s model is recommended for examinees whose performance on measures of crystallized intelligence is likely to be negatively affected by a language or hearing impairment.
c. Incorrect See explanation for response b.
d. Incorrect See explanation for response b.

The correct answer is: Luria’s

225
Q

The PTA votes to remove certain controversial books from the school library. Even though these books had been rarely checked out by students prior to the vote, once the books have been removed, teachers notice that many of their students have purchased and are reading the books. The students’ behavior is an example of:
Select one:

A.
cognitive dissonance.

B.
psychological reactance.

C.
the boomerang effect.

D.
deindividuation.

A

In this situation, the students are doing the opposite of what the PTA members want them to do.
a. Incorrect Cognitive dissonance refers to a state of discomfort that results when a person has inconsistent attitudes and behaviors.

b. CORRECT Psychological reactance occurs when a person feels his or her personal freedom is being threatened and, as a result, does the opposite of what is required or requested.
c. Incorrect The boomerang effect refers specifically to attitude change and involves changing one’s attitude in a way that is opposite to that contained in a persuasive communication. Because the boomerang effect refers to attitude change (rather than behavior change), answer b is a better response.
d. Incorrect Deindividuation occurs when a person feels anonymous because of his participation in a group. It can cause a person to act in uncharacteristic ways.

The correct answer is: psychological reactance.